Sei sulla pagina 1di 21

DIGESTIVO

1ª Vuelta CTO Medicina

1. ¿En cuál de los siguientes trastornos esperaría encontraru- esternal con la deglución y disfagia progresiva. ¿Cuál de
na disfagia orofaríngea?: lassiguientes pruebas diagnósticas debería solicitar a conti-
nuación?:
1) Enfermedad de Parkinson.
2) Esófago de Barrett. 1) Ecografía abdominal.
3) Acalasia. 2) Esofagograma con bario.
4) Anillo de Schatzki. 3) Endoscopia.
5) Hernia hiatal. 4) pHmetría.
5) Test de Bernstein.
2. Señale cuál es el tumor que con más frecuencia da lugar
auna acalasia secundaria: 8. Un paciente de 63 años ha sido estudiado en un hospital
pordolor torácico, y en una endoscopia digestiva alta con
1) Carcinoma epidermoide de esófago. tomade biopsias le han diagnosticado de esófago de Barrett,
2) Adenocarcinoma gástrico. sinobservarse displasia. Le consulta a usted para que le
3) Carcinoma de pulmón. acon-seje sobre su enfermedad, ¿cuál de las siguientes
4) Linfoma. considera-ciones NO sería correcta en este paciente?:
5) Carcinoma de páncreas.
1) Debería hacerse usted endoscopia y biopsias cada 1,5-2
3. Una de las siguientes NO suele verse como complicación años.
dela acalasia. Señálela: 2) Dado el cuadro clínico que presenta el paciente, deberíaus-
ted tomar omeprazol.
1) Disminución de peso. 3) Su enfermedad se debe probablemente al reflujo gastro-
2) Déficit de vitamina B12. esofágico.
3) Carcinoma esofágico. 4) Tiene usted riesgo de adenocarcinoma de esófago, sólosi
4) Aspiración pulmonar. existe colonización por H. pylori.
5) Esofagitis. 5) El omeprazol probablemente no le revertirá el esófago deBa-
rrett.
4. Mujer de 48 años con diagnóstico de acalasia que, trassome-
terse a dos dilataciones neumáticas, presenta recu-rrencia 9. Señale la afirmación correcta en relación al carcinoma deesó-
de su disfagia. ¿Cuál de los siguientes tratamientossería fago:
considerado de elección?:
1) Histológicamente es un adenocarcinoma en menos del10%
1) Miotomía de Heller. de los casos.
2) Resección esofágica. 2) La incidencia es similar en todas las regiones del mundo.
3) Toxina botulínica. 3) Suelen diagnosticarse precozmente por la disfagia.
4) Nitritos orales. 4) La presencia de una úlcera en el esofagograma descartael
5) Funduplicatura laparoscópica. diagnóstico de carcinoma.
5) Actualmente la supervivencia a los 5 años es del 50%.
5. Una mujer de 42 años consultó por dolor precordial irradia-
do a espalda. Aportaba un extenso estudio 10. Señale cuál es el mejor método para el estadiaje T de
cardiológico,incluyendo una coronariografía que era nor- lostumores esofágicos:
mal. Un estudioradiológico con bario y una esofagogastros-
copia fueronnormales. Se realizó una manometría que de- 1) Endoscopia.
mostró, tras ladeglución, contracciones simultáneas de gran 2) Radiología con contraste.
amplitud enla parte inferior del esófago; la presión basal del 3) Ultrasonografía endoscópica.
esfínteresofágico inferior estaba ligeramente aumentada, 4) TC.
pero serelajaba durante la deglución. ¿Qué diagnóstico 5) Laparoscopia.
Preguntas TEST

inicial haríaa esta paciente?:


11. Es cierto que, en la infección por H. pylori:
1) Acalasia cricofaríngea.
2) Acalasia vigorosa. 1) La bacteria se identifica en la lámina propia de la mucosa.
3) Espasmo esofágico difuso. 2) Se asocia siempre a colonización de la mucosa duodenal.
4) Probable amiloidosis del esófago. 3) El cultivo es la prueba diagnóstica más sensible.
5) Anillo esofágico tipo B. 4) La tasa de portadores sanos es muy elevada.
5) Se visualiza al microscopio óptico como Gram negativoy
6. Si en el estudio de un paciente se encuentra con aperistal- flagelado.
sisen los dos tercios inferiores del esófago con hipotensión
delesfínter esofágico inferior, ¿qué enfermedad explicaría 12. El aumento plasmático de la gastrina-17 en un paciente
mejorel cuadro?: congastritis crónica atrófica suele deberse en primer lugar a:

1) Acalasia clásica. 1) Asociación a carcinoma gástrico.


2) Hernia hiatal paraesofágica. 2) Gastritis por alcohol.
3) Síndrome de Boerhaave. 3) Tratamiento con sucralfato.
4) Síndrome de Sjögren. 4) Desarrollo de un tumor carcinoide.
5) Esclerodermia. 5) Presencia de hipoclorhidria.

7. Varón de 62 años, con antecedente de pirosis de largaevo- 13. Mujer de 46 años, sin antecedentes de interés, que consul-
lución y, ocasionalmente, regurgitación de alimentos. Con- tapor un episodio mensual de migraña que cede en 24
sulta porque, desde hace tres meses, nota dolor retro- horascon 2 dosis de ibuprofeno. No presenta síntomas

1
DIGESTIVO
CTO Medicina 1ª Vuelta
dispépticos y se decide continuar con el mismo tratamiento 19. Para confirmar la erradicación de H. pylori, es correctoafir-
analgé-sico. En relación con la profilaxis de la úlcera péptica mar que:
porAINEs, sería adecuado recomendar:
1) Debe suspenderse previamente la toma de omeprazol.
1) Sucralfato. 2) Es de elección el cultivo de la mucosa antral.
2) Misoprostol. 3) El test del aliento es poco sensible.
3) Famotidina. 4) Sólo está indicada ante la sospecha de resistenciabacteriana.
4) Lansoprazol. 5) Debe realizarse siempre en las úlceras duodenales.
5) Ninguna medida farmacológica.
20. En caso de ulcus gástrico, es correcto afirmar que:
14. Mujer de 52 años, recientemente diagnosticada de
Addison,que es remitida a consulta por presentar títulos 1) Se debe solicitar gastrina basal.
muy eleva-dos de anticuerpos anticélula parietal gástrica. 2) La biopsia de las zonas más atróficas es útil para demostrarH.
Actualmentese encuentra asintomática. La Hb es de 9,6 g/dl pylori.
y el volumencorpuscular es de 113 fl. Con estos datos, ¿qué 3) La erradicación de H. pylori precisa 14 días de tratamiento.
aconsejaríaa la paciente?: 4) Las localizadas en fundus suelen necesitar cirugía.
5) Debe realizarse mantenimiento con un fármacoantisecretor.
1) Realizar una gastroscopia y solicitar B12.
2) Un estudio baritado gastroduodenal. 21. Un paciente de 76 años, con antecedente de ulcus gástrico-
3) Repetir la determinación de los Ac anticélula parietal. sangrante por H. pylori al que se le erradicó con éxito
4) Tratamiento inmediato con ácido fólico. lainfección, consulta porque se le ha prescrito piroxicam
5) Revisiones periódicas, ya que la anemia se corregirá conel poruna artritis traumática de rodilla. ¿Cuál de las siguien-
tratamiento para el Addison. tesmedidas de gastroprotección recomendaría a continua-
ción?:
15. A un varón de 46 años se le remite a consultas para realiza-
runa gastroscopia por dispepsia refractaria a tratamiento- 1) Acexamato de Zinc.
con ranitidina. En la analítica se observa Hto normal y 2) Omeprazol oral.
ligerahipoalbuminemia con VSG normal. El estudio endos- 3) No precisa ninguna.
cópicodemuestra en fundus gástrico pliegues muy 4) Ranitidina-bismuto al menor síntoma.
engrosados.Con estos datos, ¿cuál sería el diagnóstico más 5) Sucralfato, y si hay síntomas, añadir misoprostol.
probable?:
22. En un varón de 56 años, sin antecedentes de interés, excep-
1) Adenocarcinoma gástrico tipo intestinal. totabaquismo moderado, se le ha prescrito ibuprofeno por-
2) Gastritis crónica tipo A. dolor articular. ¿Cuál de las siguientes medidas es la má-
3) Gastritis eosinofílica. sadecuada?:
4) Ménètrier.
5) Gastritis aguda por H. pylori. 1) Parches de nicotina, para evitar riesgo de ulcus.
2) Misoprostol, 3-4 veces al día.
16. ¿Cuál de los siguientes AINEs tiene menor potencial gastro- 3) Administrar rofecoxib en vez de ibuprofeno.
erosivo?: 4) Recomendar dieta rica en lácteos.
5) Ranitidina y magaldrato, si tiene dispepsia.
1) Meloxicam.
2) Diclofenaco. 23. Señale cuál de los siguientes NO es un factor predisponen-
3) Rofecoxib. tede cáncer gástrico:
4) Ibuprofeno.
5) Nabumetona. 1) Esófago de Barrett.
2) Infección por H. pylori.

Preguntas TEST
17. Señale la correcta: 3) Síndrome de Lynch tipo II.
4) Anemia perniciosa.
1) La úlcera gástrica se perfora con más frecuencia que laduo- 5) Alcoholismo crónico.
denal.
2) Un 10% de las úlceras duodenales recidivan. 24. En un enfermo con diarrea y fiebre de 7 días de
3) Las úlceras duodenales suelen ser más grandes que lasgástricas. evolución,¿cuál de las siguientes pruebas diagnósticas indi-
4) Las úlceras duodenales aparecen sobre todo en la primera- caría enprimer lugar?:
porción del duodeno.
5) Las úlceras del fundus siempre son benignas. 1) Sigmoidocolonoscopia.
2) Leucocitos fecales.
18. En el tratamiento de un paciente con ulcus duodenal san- 3) Grasas fecales.
grante y test de la ureasa positivo, son correctas las siguien- 4) Coprocultivo.
tes decisiones, EXCEPTO: 5) Tránsito intestinal.

1) Test del aliento para H. pylori, para confirmar la erra-dicación. 25. En la enfermedad celíaca, NO es un hallazgo característico:
2) Tratar con metronidazol, si hay alergia a amoxicilina.
3) Tratamiento con bismuto coloidal, si presentaintolerancia a 1) Los anticuerpos IgA antitransglutaminasa.
claritromicina. 2) Asociación a HLA B51.
4) Cultivo de la mucosa antral para H. pylori, si hay fracasoste- 3) Hiperplasia críptica.
rapeúticos. 4) Infiltración linfoplasmocitaria de la lámina propia.
5) Evitar biopsias de los bordes de la úlcera. 5) Linfocitos intraepiteliales.

2
DIGESTIVO
1ª Vuelta CTO Medicina

26. ¿Cuál le parece la causa más frecuente de que un paciente- 1) Sigmoidoscopia.


con esprue celíaco continúe con síntomas tras iniciar trata- 2) Colonoscopia.
miento?: 3) Enema opaco.
4) Gastroduodenoscopia.
1) Linfoma. 5) Arteriografía.
2) Mala realización de la dieta.
3) Esprue colágeno. 33. La presencia de anemia ferropénica y melenas en una
4) Neoplasia esofágica. mujerque tiene 23 años, sugiere fundamentalmente el diag-
5) Error diagnóstico. nósticode:

27. Entre las siguientes, es una causa de diarrea secretora: 1) Ulcus duodenal.
2) Divertículo de Meckel.
1) Ingesta de hidróxido de magnesio. 3) Cáncer de colon.
2) Síndrome del intestino irritable. 4) Poliposis rectal.
3) Administración de lactulosa. 5) Adenocarcinoma gástrico.
4) Síndrome carcinoide.
5) Diarrea postvagotomía con piloroplastia. 34. Varón de 16 años, con antecedentes de 2 ó 3 episodiosanua-
les de dolor en fosa ilíaca derecha, que duran entre 2y 5 días.
28. Las siguientes enfermedades pueden cursar con entero- Habitualmente se acompañan de diarrea y seresuelven de
patía y pérdida de proteínas, EXCEPTO: forma espontánea. Consulta porque, en elúltimo episodio,
el dolor es más intenso, se acompaña defiebre y tres depo-
1) Carcinoma esofágico. siciones al día. En la exploración física seencuentra con buen
2) Pericarditis constrictiva. estado general. En la analítica destacaleucocitosis. El copro-
3) Enfermedad de Whipple. cultivo es negativo. En una sigmoido-colonoscopia se apre-
4) Colitis granulomatosa. cian úlceras aftoides en ciego y colonderecho. En las biop-
5) Hipertiroidismo. sias se aprecia un infiltrado inflamatorioinespecífico. ¿Cuál,
de entre las siguientes opciones tera-péuticas, considera la
29. En el sobrecrecimiento bacteriano, es cierto todo lo si- más apropiada para este paciente?:
guiente, EXCEPTO:
1) Antagonistas de TNF.
1) La esteatorrea se debe a desconjugación de las salesbiliares. 2) Esteroides.
2) Los test del aliento son útiles para el diagnóstico. 3) Metronidazol y mesalazina.
3) Suele haber recurrencias. 4) Azatioprina.
4) Puede haber atrofia intestinal leve. 5) 6-Mercaptopurina.
5) Es excepcional en los gastrectomizados.
35. Señale la correcta:
30. Un varón de 14 años consulta por diarrea crónica. Presen-
taascitis y edemas en miembros inferiores, más intensos 1) Puede haber ANCA positivos en la colitis ulcerosa.
sobretodo en el miembro inferior derecho. En los análisis 2) La colitis ulcerosa es más frecuente en fumadores.
aparecehipoalbuminemia y linfopenia. La serología de VIH 3) En el 50% de las personas con colitis ulcerosa apareceuna
es nega-tiva, no hay proteinuria y el aclaramiento de alfa-1- colangitis esclerosante.
antitrip-sina está aumentado. La paracentesis muestra una 4) La colitis ulcerosa afecta siempre y de forma uniforme atodo
ascitisquilosa. Se realiza una biopsia intestinal que confirma el colon.
eldiagnóstico clínico de: 5) Es característico de la colitis ulcerosa la presencia dehemorra-
gias sin moco.
1) Ileítis tuberculosa.
2) Enfermedad de Whipple. 36. Una paciente de 32 años consulta por presentar, desde
Preguntas TEST

3) Esprue refractario. 2semanas antes, diarrea sanguinolenta de 6 deposiciones-


4) Giardiasis. diarias, dolor abdominal y febrícula. Dos coprocultivos fue-
5) Linfangiectasia intestinal. ron negativos. Una colonoscopia demostró una mucosaeri-
tematosa y granular con hemorragias puntuales desderecto
31. Varón de 80 años, con antecedente de dos episodios dehe- a ángulo esplénico del colon. La biopsia fue compatiblecon
morragia digestiva alta por úlcera duodenal hace 30 años,y el diagnóstico de enfermedad inflamatoria crónica intes-
estenosis aórtica con insuficiencia cardíaca grado II.Consulta tinal. ¿Cuál de las siguientes actitudes sería más correcta?:
por presentar, hace diez días, rectorragias que leduraron
dos días. Actualmente continúa asintomático. Unasigmoido- 1) Mesalazina y metronidazol.
colonoscopia hasta ángulo esplénico no demues-tra lesio- 2) Sulfasalazina sola.
nes. ¿Qué medida diagnóstica recomendaría a con-tinua- 3) Esteroides y metronidazol.
ción?: 4) Mesalazina y esteroides.
5) Ciprofloxacino y mesalazina.
1) Nueva colonoscopia hasta ciego.
2) Arteriografía. 37. Uno de los siguientes hallazgos es más frecuente en la
3) Gastroduodenoscopia. colitisulcerosa que en la enfermedad de Crohn. Señálelo:
4) Estudio baritado gastroduodenal. 1) Dolor abdominal.
5) Enema opaco. 2) Pérdida de peso.
3) Masa abdominal.
32. ¿Cuál de las siguientes pruebas realizaría en primer lugar- 4) Friabilidad al estudio endoscópico.
tras una hematoquecia?: 5) Fístulas.

3
DIGESTIVO
CTO Medicina 1ª Vuelta
38. En el tratamiento de la enfermedad de Crohn, las siguientes- 45. Un varón de 66 años, sin antecedentes personales ni fami-
pautas son adecuadas, EXCEPTO en el caso de: liares de interés, consulta por hematoquecia. En una colo-
noscopia se le extirpan 3 pólipos adenomatosos sin signos-
1) Esteroides, como mantenimiento tras brote leve-mo-derado, de displasia en sigma y colon izquierdo. En relación con
ya que garantizan una remisión prolongada. elcribado del cáncer de colon, sería lo más apropiado some-
2) Azatioprina, ante enfermedad crónicamente activa. terperiódicamente al paciente a:
3) En brotes graves, puede indicarse ciclosporina i.v.
4) Colestiramina puede usarse en casos de afectación ilealseve- 1) Test de hemorragias ocultas.
ra, para control de la diarrea. 2) Colonoscopia.
5) Sulfasalazina es eficaz en la colitis activa. 3) Eco hidrocólica.
4) Niveles de antígeno CEA.
39. Una mujer de 35 años, diagnosticada de colitis ulcerosa 5) Si está asintomático, solamente revisiones en consulta.
4años antes, ingresa con fiebre, distensión abdominal dolo-
rosa, diarrea de múltiples deposiciones y signos de deshi- 46. En la aparición de un carcinoma colorrectal, NO se conside-
dratación. Presenta leucocitosis y anemia. ¿Qué prueba- raun factor importante la existencia previa de:
diagnóstica realizaría en primer lugar?:
1) Antecedentes familiares.
1) Colonoscopia. 2) Síndrome del colon irritable.
2) Ecografía abdominal. 3) Colitis ulcerosa.
3) Enema opaco. 4) Poliposis colorrectal.
4) Radiografía de abdomen. 5) Síndrome de los adenomas planos.
5) TC abdominal.
47. En relación a la asociación entre el adenocarcinoma decolon
40. En la colitis isquémica subaguda, es cierto que: y la colitis ulcerosa, señale la que es INCORRECTA:

1) Se produce por la obstrucción proximal de la arteriacólica 1) El riesgo depende de la extensión de la enfermedad.


izquierda. 2) El riesgo se relaciona con la duración de la enfermedad.
2) El recto está afectado en el 80% de los casos. 3) El riesgo se relaciona con el grado de displasia.
3) La angiografía es la técnica diagnóstica de elección. 4) Se recomienda colectomía profiláctica a los 15 años deevo-
4) Las imágenes “en huella dactilar” la diferencian de laforma lución de la enfermedad.
aguda. 5) Existe mayor riesgo cuando la enfermedad se presenta aeda-
5) Los síntomas remiten de forma habitual en 3 ó 4 semanasy no des más jóvenes.
recidivan.
48. La presencia de cáncer de colon en varios miembros de
41. Ante la sospecha de una diverticulitis complicada con unabsce- unafamilia, a edades inferiores a los 50 años y sin evidencia
so de pared, solicitaría como prueba diagnóstica másrentable: depoliposis asociada, le sugiere un síndrome de:

1) TC. 1) Lynch tipo I.


2) Eco abdominal. 2) Gardner.
3) Rx lateral de abdomen. 3) Muir-Torre.
4) Enema opaco. 4) Cowden.
5) Ecografía hidrocólica. 5) Peutz-Jeghers.

42. En relación a la hemorragia diverticular, es FALSO: 49. En el seguimiento de un paciente intervenido de carcinoma-
de colon estadio C, la elevación del antígeno CEA significa:
1) Suele originarse en el colon derecho.
2) Es una complicación poco frecuente de la enfermedaddiver- 1) Recidiva tumoral.

Preguntas TEST
ticular del colon. 2) Indicación de test de hemorragias ocultas.
3) En la mayoría de los casos, la hemorragia recidiva. 3) Error inicial en el estadiaje.
4) Suele presentarse como hematoquecia indolora. 4) Síndrome de Lynch.
5) Suele dejar de sangrar espontáneamente. 5) Una segunda neoplasia a nivel rectal.

43. Señale cuál de entre las siguientes características de lospó- 50. Mujer de 46 años, con antecedente de enfermedad celía-
lipos de colon NO se correlaciona con el potencial demalig- caen la adolescencia que consultó por presentar diarreaa-
nización: cuosa de 3 meses de evolución. En ocasiones se acompa-
ñaba de moco, pero no de sangre ni pus. Un test de
1) El tamaño. grasasfecales fue de 3 g/día. Los coprocultivos y estudios
2) El tipo histológico. deparásitos fecales fueron negativos. Los anticuerpos an-
3) El grado de displasia. tiglia-dina y antiendomisio y los ANCA fueron negativos.
4) La localización. Lasbiopsias de duodeno y yeyuno mostraron una mucosa-
5) La multiplicidad. normal. En una posterior colonoscopia se observó unamu-
cosa ligeramente enrojecida en sigma y colon izquierdo.En
44. El síntoma más común de los pólipos colorrectales es: el estudio histológico se demostró un infiltrado predomi-
nantemente crónico con linfocitos y plasmáticas, sin micro-
1) Intususcepción. abscesos ni granulomas. ¿Cuál de los siguientes diagnós-
2) Prolapso. ticoses el más probable?:
3) Rectorragia.
4) Estreñimiento. 1) Síndrome de intestino irritable.
5) Diarrea. 2) Colitis ulcerosa.

4
DIGESTIVO
1ª Vuelta CTO Medicina

3) Celíaca colágena. laecografía confirmó la presencia de líquido libre peritoneal.El


4) Hipertiroidismo. estudio del líquido ascítico demostró un gradiente albúmi-
5) Colitis linfocítica. na sérica/albúmina en ascitis de 3,1. ¿Cuál de estos diagnós-
ticos sería MENOS probable?:
51. Un paciente de 44 años consultó por rectorragias. Unacolo-
noscopia demostró un cáncer de sigma y múltiplespólipos 1) Síndrome de Budd-Chiari.
adenomatosos en todo el colon. A su hermana de 47años se 2) Cirrosis biliar primaria.
le realizó una sigmoidoscopia que fue normal. ¿Quétipo de 3) Tuberculosis peritoneal.
seguimiento habría que hacerle a la hermana?: 4) Enfermedad venooclusiva hepática.
5) Trombosis de la porta.
1) Rectosigmoidoscopia cada 2 años.
2) Colonoscopia cada 3 años. 58. En cuál de los siguientes casos de ascitis maligna sueleen-
3) Enema opaco anual. contrarse citología positiva en líquido ascítico:
4) Ningún seguimiento.
5) Proponerle colectomía, aunque no tenga pólipos, yendosco- 1) Carcinomatosis peritoneal, sin metástasis hepáticas.
pia alta cada 2 años, posteriormente. 2) Metástasis masivas hepáticas, sin carcinomatosis perito-neal
asociada.
52. ¿Cuál de las siguientes alteraciones NO se asocia con unau- 3) Ascitis quilosa de origen tumoral.
mento aislado de la bilirrubina indirecta sérica?: 4) Metástasis masivas hepáticas, con gran aumento de lafosfata-
sa alcalina.
1) Ictericia fisiológica del recién nacido. 5) Pacientes con aumento de la alfafetoproteína en líquidoascí-
2) Ictericia por lactancia materna. tico.
3) Ictericia hemolítica.
4) Síndrome de Gilbert. 59. Señale la respuesta FALSA en relación a la peritonitisbacte-
5) Síndrome de Dubin-Johnson. riana espontánea:

53. Una paciente de 29 años consulta por padecer astenialeve. 1) Está producida generalmente por gramnegativos.
En la exploración física se aprecia una dudosa ictericiay el 2) Cursa con peritonismo en el 80% de los casos.
resto es normal. Se realiza una analítica de orinaurgente que 3) En el tratamiento empírico, no es necesario cubriranaerobios.
demuestra dos cruces de bilirrubina. ¿Cuálde los siguientes 4) Su tratamiento no incluye la cirugía.
trastornos sería MENOS probable?: 5) El parámetro más utilizado para el diagnóstico deperitonitis
bacteriana espontánea es la cifra de polimor-fonucleares.
1) Coledocolitiasis.
2) Ampuloma. 60. Indicaría la profilaxis antibiótica con quinolonas de laperito-
3) Hepatitis. nitis bacteriana espontánea en un cirrótico con ascitis:
4) Colestasis benigna recurrente.
5) Síndrome de Gilbert. 1) Si asocia antecedentes de alcoholismo.
2) En caso de hemorragia digestiva alta.
54. ¿Cuál es la primera exploración instrumental que deberea- 3) Únicamente a partir de haber padecido un segundoepisodio
lizarse en un paciente con colestasis?: de peritonitis.
4) Cuando las proteínas en líquido ascítico son >1 g/dl.
1) Colecistografía oral. 5) Cuando los neutrófilos en líquido ascítico son <100/mm3.
2) Colangiografía intravenosa.
3) Colangiografía retrógrada endoscópica. 61. ¿Cuál de los siguientes datos histológicos renales es típico-
4) Ecografía. del síndrome hepatorrenal?:
5) TC.
1) Proliferación mesangial.
Preguntas TEST

55. Señale cuál de las siguientes NO es una característica 2) Trombos de bilirrubina en las asas de Henle.
delsíndrome alcohol-paracetamol: 3) Infiltración intersticial aguda de linfocitos.
4) Trombosis de la vena renal.
1) Puede producir un fallo hepático agudo. 5) Histología normal.
2) Está descrito incluso con el empleo de dosis terapéuticasde
paracetamol. 62. Paciente de 45 años, en tratamiento con espironolactona
3) Tiene un buen pronóstico, en relación al fallo hepáticoagudo porascitis secundaria a cirrosis en estadio C, y con varios
en general. episodiosde sangrado por varices esofágicas, por lo que se
4) Es una de las causas más frecuentes de fallo hepáticoagudo. encuentra ala espera de trasplante. Reingresa por un nuevo
5) Las transaminasas no suelen estar muy elevadas. episodio dehematemesis. En una endoscopia se observan
varices grado IVcon puntos sangrantes. Tras 48 horas de
56. El tratamiento de elección de una ascitis a tensión es: iniciar el tratamientocon somatostatina y esclerosis de las
varices, continúa sangrando con descenso notable de la TA
1) Dieta hiposódica y de restricción hídrica. y del hematócrito. Lassiguientes medidas podrían estar
2) Espironolactona 100 mg/día. indicadas, EXCEPTO:
3) Paracentesis evacuadora y reposición con albúmina.
4) Espironolactona asociado a furosemida. 1) Quinolonas orales.
5) Colocación de TIPS. 2) Colocación de TIPS.
3) Administrar lactulosa en enemas.
57. Un paciente de 47 años consultó por molestias abdominale- 4) Transfundir concentrados de hematíes.
sinespecíficas y aumento del perímetro abdominal. En laex- 5) Aumentar la dosis del diurético.
ploración física se detectó matidez en ambos flancos, y

5
DIGESTIVO
CTO Medicina 1ª Vuelta
63. A un varón de 44 años, asintomático, se le realizó una 1) Infección aguda por VHB.
biopsiahepática por hepatitis VHC. El estudio histológico 2) Infección crónica por VHB.
revelócomo hallazgo una cirrosis. Una ecografía abdominal 3) Infección aguda por VHA.
y laendoscopia oral fueron normales. En relación con la 4) Probable sobreinfección delta.
profi-laxis de la hemorragia por varices esofágicas, ¿qué 5) Hepatitis no vírica.
cree lomás adecuado recomendar?:
70. Un paciente vacunado frente al VHB hace 2 años y conres-
1) Nitritos en dosis bajas. puesta desconocida a la vacuna sufre un contacto acci-
2) Propranolol. dental con el VHB. Su título de anti-HBs es <10 mUI/ml.
3) Ligadura endoscópica. ¿Quéle recomendaría?:
4) Únicamente vigilancia periódica.
5) Losartán. 1) Vacunación completa de nuevo.
2) Una dosis de Ig y una dosis de recuerdo de vacuna.
64. Señale la respuesta INCORRECTA en relación a la infec- 3) Dos dosis de Ig.
ciónpor el VHC: 4) Ningún tratamiento.
5) Una dosis de recuerdo de vacuna.
1) El ARN-VHC es el mejor marcador para el diagnóstico.
2) Tiene un riesgo elevado de evolución a la cronicidad. 71. Señale el dato histológico mínimo para poder hacer eldiag-
3) Es frecuente en drogadictos parenterales. nóstico de hepatitis crónica activa:
4) La mayoría tienen historia de transfusión.
5) Un porcentaje importante de casos no tienen unantecedente 1) Necrosis hepática con puentes.
epidemiológico de interés. 2) Colapso multilobular.
3) Nódulos de regeneración.
65. Uno de los siguientes marcadores serológicos es impres- 4) Necrosis parcelar periférica.
cindible para diagnosticar una infección aguda por VHB: 5) Infiltrado mononuclear del espacio porta.

1) HBsAg. 72. Un paciente de 43 años consulta por molestias abdomina-


2) ADN-VHB. les inespecíficas. En la exploración física presenta única-
3) Anti-HBc de tipo IgM. mente un discreto aumento del tamaño hepático. Los datos-
4) HBeAg. de laboratorio ofrecen los siguientes resultados: GPT 100UI/
5) Actividad ADN-polimerasa. l, GOT 87 UI/l, HBsAg+, HBeAg-, anti-HBc+ de tipo IgG,anti-
HBe+, ADN-VHB+. El resto de los marcadores víricosdieron
66. Señale cuál de las siguientes manifestaciones extrahepáticas- resultado negativo. ¿Qué diagnóstico haría?:
de la infección por el virus de la hepatitis C es más frecuente:
1) Infección crónica por VHB en fase no replicativa.
1) Crioglobulinemia mixta esencial. 2) Infección aguda por VHB.
2) Glomerulonefritis membranoproliferativa. 3) Infección crónica por la cepa mutante precore.
3) Glomerulonefritis membranosa. 4) Infección pasada por VHB.
4) Anemia aplásica. 5) Probable hepatopatía tóxica o metabólica. Se puededescar-
5) Polineuropatía periférica. tar enfermedad vírica.

67. ¿En cuál de las siguientes situaciones existe mayor riesgo 73. En relación con el empleo de lamivudina en el tratamientode
decronificación de la infección por VHB?: la hepatis crónica VHB, es cierto que:

1) Adquisición por vía parenteral. 1) Está contraindicada en VIH positivos.


2) Tras contagio sexual. 2) Está indicado en la mutante Ag e negativo.
3) Adquisición en la infancia. 3) Su efecto adverso más frecuente es el hipotiroidismo.

Preguntas TEST
4) Contagio en la edad adulta. 4) La vía de administración es intramuscular semanal.
5) Tras contagio concomitante con infección por VIH. 5) Debe usarse asociado a interferón.

68. Un paciente de 28 años, diagnosticado de infección crónica- 74. ¿Cuál es el objetivo principal del tratamiento de la hepati-
por VHB en fase no replicativa, sufre un cuadro de hepatiti- tiscrónica por VHB?:
saguda. Los datos de laboratorio confirman el diagnóstico-
clínico. Los marcadores víricos dan los siguientes resulta- 1) La eliminación del virus del organismo.
dos: HBsAg+, HbeAg-, anti-HBc+, ADN VHB-, anti-VHC- 2) La seroconversión anti-e.
,HD-Ag+, anti-HD IgM-, anti-HD IgG- e IgM anti-VHA-. 3) Que el virus, aunque se replique, no produzca lesiónhepática.
¿Quédiagnóstico haría?: 4) Evitar la aparición de mutantes precore y de escape.
5) Mejorar la cirrosis hepática.
1) Reactivación de la infección por VHB.
2) Seroconversión anti-e. 75. A una mujer de 37 años, ex ADVP, se le descubren anticuer-pos
3) Sobreinfección delta. anti-VHC positivos. Durante seis meses de seguimiento,las tran-
4) Coinfección delta. saminasas y el resto de la bioquímica hepática fueron normales.
5) No hay datos para poder asegurar un diagnóstico concerteza. El RNA VHC fue +. En el manejo de la paciente, recomendaría:

69. Un paciente de 23 años, ADVP activo, consulta por uncuadro 1) Biopsia hepática.
clínico y bioquímico compatible con una hepatitisaguda. 2) Interferón.
Presenta los siguientes marcadores: HBsAg-, anti-HBs-, anti- 3) Solicitar anticuerpos antimitocondriales.
HBc+, anti-HBe+, anti-VHC-, IgM anti-VHA+.¿Qué diag- 4) Interferón + ribavirina.
nóstico haría?: 5) Seguimiento evolutivo.

6
DIGESTIVO
1ª Vuelta CTO Medicina

76. En relación con la terapia antiviral de la hepatitis crónica 82. Señale la opción FALSA en relación a la colangitis esclero-
porVHB, es cierto todo lo siguiente, EXCEPTO: sante:

1) El interferón gamma es el fármaco más eficaz. 1) Existe asociación con el HLA-B8.


2) La respuesta es inferior en los VIH positivos. 2) La mayoría de los casos aparecen en pacientes quetienen
3) La respuesta es superior en las mujeres. colitis ulcerosa.
4) Los niveles elevados de ALT predicen una buenarespuesta. 3) La colectomía en la colitis ulcerosa protege del desarrollode
5) Los pacientes con niveles bajos de ADN-VHB respondenmejor. colangitis esclerosante.
4) El diagnóstico se establece basándose en los hallazgoscolan-
77. Respecto a un paciente de 62 años, con cirrosis posthe- giográficos.
patitis por VHC, al que se va a someter a un trasplantehepá- 5) Es más frecuente en varones.
tico, es correcto afirmar que:
83. El hijo de 26 años de un paciente afecto de hemocromatosis-
1) Es muy probable que el nuevo hígado se infecte por VHC. presenta un índice de saturación de transferrina muy elevado
2) En un alto porcentaje, se detecta anti-VHC. (65 %) y ferritina sérica de 550 (N < 200). A continuación,se
3) La presencia de crioglobulinas es una indicación inclusoen debe aconsejar:
estadio A de Child.
4) No puede ser trasplantado por la edad. 1) Biopsia hepática.
5) Si el genotipo es Ib, es menor el riesgo de recidiva precozpos- 2) Estudio mutaciones gen HFE.
transplante. 3) Flebotomías.
4) D-penicilamina.
78. Una paciente de 18 años, sin antecedentes de interés,consulta 5) TC abdominal.
por hipertransaminasemia persistente. En el estu-dio inmu-
nológico se detectan ANA y ANCA positivos conmarcadores 84. Señale cuál de las siguientes es correcta, en la hemocroma-
víricos negativos. Ante esta situación, es correc-to afirmar tosis primaria:
que:
1) El hipogonadismo puede ser el cuadro de presentaciónen
1) Presenta una colangitis esclerosante. jóvenes.
2) Se trata de una hepatitis crónica autoinmune tipo 2. 2) La intolerancia hidrocarbonada mejora con las fle-botomías.
3) La biopsia hepática no establecerá el diagnóstico decerteza. 3) El gen anómalo está en el cromosoma 12.
4) Hay que descartar una enfermedad de Wilson. 4) El hipogonadismo se debe al acúmulo de hierro en lasgónadas.
5) Debe recibir tratamiento con tacrolimus. 5) Cursa con herencia autosómica dominante.

79. A un paciente de 51 años con cirrosis grado C y frecuentese- 85. ¿Cuál de las siguientes entidades NO se asocia de formaha-
pisodios de encefalopatía y ascitis, se le aprecia una lesión- bitual con esteatosis hepática?:
de 3 cm en lóbulo hepático derecho. Una PAAF confirma
quese trata de un hepatocarcinoma. ¿Cuál de las siguiente- 1) Cirrosis alcohólica.
sopciones estaría indicada?: 2) Hepatitis crónica VHB mutante precore.
3) Obesidad.
1) Trasplante. 4) Diabetes mellitus.
2) Embolización. 5) Tratamiento con corticoides.
3) Alcoholización percutánea.
4) Cirugía. 86. ¿En cuál de las siguientes enfermedades existe mayor ries-
5) Cisplatino y adriamicina. gode desarrollar cálculos pigmentarios en la vesícula biliar?:

80. Una paciente de 38 años consultó por prurito y astenia.En la 1) Insuficiencia renal crónica.
exploración física presentaba una hepatomegalialisa a 2 2) Estado de portador crónico del VHB.
Preguntas TEST

centímetros del reborde costal. Los datos delaboratorio 3) Enfermedad de Crohn.


fueron: bilirrubina 1,6 mg/dl; fosfatasa alcali-na 1.040 U/L 4) Estados hemolíticos crónicos.
(normal <280 U/L); el resto de la bioquímicahepática era 5) Obesidad.
normal; los anticuerpos antimitocondrialesfueron positivos.
Una ecografía abdominal fue normal.¿Cuál de los siguientes 87. Señale la causa por la que la bilis se vuelve más litogénica
fármacos sería más útil para lapaciente?: traslas resecciones ileales:

1) Interferón. 1) Se reabsorbe menos agua.


2) Penicilamina. 2) Se pierde colesterol por la diarrea.
3) Trientine. 3) Disminuye la reabsorción de sales biliares.
4) Esteroides. 4) Aumenta la reabsorción de colesterol.
5) Ácido ursodesoxicólico. 5) El flujo biliar es más lento.

81. ¿Cuál de estos parámetros tendría MENOS valor a la hora 88. La ecografía abdominal en una pancreatitis aguda ofrece-
deevaluar a un paciente con cirrosis biliar primaria?: particularmente información sobre:

1) Niveles de IgM. 1) Confirmación del diagnóstico de pancreatitis.


2) Anticuerpos antimitocondriales. 2) La etiología.
3) Hormonas tiroideas. 3) El pronóstico.
4) Niveles de bilirrubina. 4) La evolución.
5) Niveles de triglicéridos. 5) Modificaciones del tratamiento.

7
DIGESTIVO
CTO Medicina 1ª Vuelta
89. ¿Cuál es la causa más frecuente de las pancreatitis agudas?: 3) Hiperlipemia.
4) Mucoviscidosis.
1) Alcoholismo. 5) Poliovirus.
2) Litiasis biliar.
3) Microlitiasis. 96. En el curso de la pancreatitis crónica, es EXCEPCIONAL
4) Esfínter de Oddi hipertensivo. quesuceda:
5) Fibrosis quística.
1) Aparición de calcificaciones pancreáticas.
90. ¿Cuál de las siguientes pruebas es la de mayor sensibilida- 2) Cetoacidosis diabética.
dpara el diagnóstico de pancreatitis crónica?: 3) Déficit de B12.
4) Ictericia.
1) Ecografía abdominal. 5) Ascitis pancreática.
2) Rx simple de abdomen.
3) TC. 97. Paciente alcohólico, de 49 años, que consulta por dolorepi-
4) CPRE. gástrico postprandial de 3 meses de evolución. En unaeco-
5) Test de secretina. grafía se observa una masa pancreática que se confirmaen
la TC. Una CPRE muestra una estenosis única en elWirsung.
91. Todos los siguientes son criterios de Ransom, EXCEPTO Con estos datos, el diagnóstico más probable es:
unode ellos. Señálelo:
1) Pancreatitis aguda cefálica.
1) Calcio < 8 mg/dl. 2) Pancreatitis crónica etílica.
2) pO2 < 60 mmHg. 3) Carcinoma de páncreas.
3) GOT>250 UI. 4) Pancreatitis crónica obstructiva.
4) Leucocitosis >16.000. 5) Nesidioblastosis.
5) PCR > 2 veces superior al límite normal.
98. El parámetro más apropiado para indicar el tratamiento
92. Para evitar complicaciones en el curso de la pancreatitisagu- conenzimas pancreáticas como substitución es:
da moderada, es necesario:
1) Test de la secretina.
1) Diferir el inicio de la alimentación oral. 2) Quimiotripsina fecal.
2) Iniciar inmediatamente tratamiento antibiótico. 3) Tripsina sérica.
3) Tratar precozmente con glucagón. 4) Grasas fecales.
4) Cirugía precoz. 5) Test del pancreolauryl.
5) Tratar con esteroides.
99. En el manejo de la ascitis pancreática, ¿cuál de entre lassi-
93. Un paciente de 33 años, alcohólico, ingresa por una pancre- guientes medidas diagnósticas es la más importante?:
atitis aguda sin evidencia de litiasis biliar. A las 24
horas,presenta fiebre, hipotensión, oliguria e hipoxemia. 1) CPRE.
Un TCcon contraste urgente demuestra un área de necrosis 2) TAC.
pan-creática del 70%, sin evidencia de absceso. Una punció- 3) Ecografía.
naspiración con aguja fina de la zona afectada demuestra- 4) Gammagrafía.
bacilos gramnegativos. Entre las siguientes, ¿qué medidate- 5) Laparoscopia.
rapéutica propondría para mejorar la evolución?:
100. Las siguientes medidas pueden ser útiles en el tratamiento-
1) Necrosectomía pancreática. del dolor en la pancreatitis crónica, EXCEPTO:
2) Somatostatina i.v.
3) Imipenem i.v. 1) Pancreatectomía cefálica.

Preguntas TEST
4) Esteroides i.v. 2) Suplementos de enzimas pancreáticos.
5) Drenaje percutáneo dirigido por TC. 3) AINEs.
4) Antagonistas del TNF.
94. Una paciente de 70 años ingresa en el hospital por dolora- 5) Dilatación y endoprótesis del Wirsung.
bdominal y vómitos. Los estudios analíticos demuestranuna
amilasa muy elevada. Una ecografía demuestra unaligera
dilatación de la vía biliar y colelitiasis. A las 48 horasde
ingreso continúa con fuerte dolor abdominal, el Hto hadis-
minuido un 15%, el calcio a 7,8 mg/dl, el BUN ha aumen-tado
15 mg/dl y la albúmina es 3,1 g/dl. ¿Cuál de los siguientestra-
tamientos sería más adecuado?:

1) Papilotomía endoscópica.
2) Pancreatectomía.
3) Administración de antibióticos, más el tratamiento habi-tual.
4) Colecistectomía urgente.
5) Colecistectomía laparoscópica.

95. ¿Cuál es la causa más frecuente de pancreatitis crónica?:

1) Alcoholismo crónico.
2) Malnutrición calórico-proteíca.

8
DIGESTIVO
1ª Vuelta CTO Medicina
Pregunta 1.-R: 1 La toxina botulínica administrada vía endoscópica en esófago
El término disfagia orofaríngea incluye a las enfermedades farín- distal se reserva para los casos de imposibilidad de someterse al trata-
geas y a las del esófago superior. Las enfermedades involucradas con miento dilatador o la miotomía. La toxina actúa paralizando a las
mayor frecuencia producen disfagia por: únicas fibras que permanecen funcionantes en el esfínter (las que lo
1. Debilidad de los músculos faríngeos, como en las miopatías. mantienen contraído permanentemente) y, por tanto, como conse-
2. Rigidez muscular, como en el Parkinson que es la respuesta correcta. cuencia de su acción el esfínter quedará constantemente relajado.
3. Alteraciones en la motilidad secundarias a origen neurógeno, bien Su desventaja es que requiere repetidas administraciones. Los nitritos
de nervio periférico o de SNC como es el caso de los ictus. orales únicamente alivian el dolor torácico, pero no son resolutivos
del problema esencial de la enfermedad.
Estas enfermedades suelen comportarse como disfagias motoras y
por tanto tienen síntomas tanto con la ingesta de sólidos como de Pregunta 5.-R: 3
líquidos. El síntoma que orienta con mayor precisión al origen orofa- La presencia en una manometría de contracciones simultáneas
ríngeo de una disfagia es la presencia de episodios de aspiración de gran amplitud y duración en el esófago distal puede deberse a dos
relacionados con la deglución (“atragantamiento”). entidades: el espasmo esofágico difuso, que es la respuesta correcta y
La acalasia es también una enfermedad motora, por tanto con la acalasia vigorosa. Se distinguen porque esta última presenta ade-
disfagia a sólidos y líquidos, pero afecta al tercio inferior del esófago más dificultad para relajar el esfínter durante la deglución. En reali-
fundamentalmente. El esófago de Barrett causa disfagia, si se asocia a dad la acalasia vigorosa no es una enfermedad o entidad clínica
una esofagitis con estenosis péptica. El anillo de Schatzki, o anillo B, como tal, sino que se trata generalmente de una acalasia en fases
es una protrusión en la zona inferior del esófago que causa disfagia iniciales de su evolución y que por la denervación de la pared esofá-
recurrente e intermitente. gica presenta contracciones de gran amplitud, por lo que además de
disfagia, puede tener episodios de intenso dolor torácico, retroester-
Pregunta 2.-R: 2 nal, y en ocasiones odinofagia.
El diagnóstico de certeza de la acalasia es manométrico. Sin em- La amiloidosis esofágica tiene un comportamiento clínico y ma-
bargo siempre debe realizarse una endoscopia con biopsias del seg- nométrico muy parecido a la esclerodermia: disfagia motora y re-
mento distal del esófago para excluir neoplasias que simulen una flujo.
acalasia, y que se conocen como acalasias secundarias. Estos tumo- El anillo B o de Schatzki causa disfagia intermitente a sólidos y
res cursan con una disfagia semejante a la acalasia primaria, porque menos frecuentemente dolor.
infiltran la pared del esófago alterando la motilidad de su zona distal La acalasia cricofaríngea o barra faríngea es una enfermedad
y esfínter. Los tumores mucosos esofágicos, como el carcinoma epi- motora que provoca disfagia orofaríngea. Debe diagnosticarse por
dermoide o el adenocarcinoma, al ser mucosos se comportan como manometría y, en ocasiones, se aprecia en la Rx cervical lateral una
una obstrucción y, por tanto, inicialmente con disfagia a sólidos y en protrusión en la región prevertebral anterior, que corresponde al
los estadíos muy avanzados, a líquidos. Entre los tumores que causan músculo cricofaríngeo contraído.
acalasia secundaria destacan:
1. El más frecuente de todos: el adenocarcinoma gástrico, que origi- Pregunta 6.-R: 5
na acalasia cuando progresa infiltrando el espesor de la pared y Los trastornos motores del esófago distal son fundamentalmente
llega a la zona distal del esófago y al esfínter esofágico inferior. Por el espasmo esofágico difuso, la acalasia y la esclerodermia. Estos dos
eso la respuesta correcta es adenocarcinoma gástrico e incluso, si últimos presentan ondas sugestivas de un peristaltismo reducido e
se hubiera expuesto, de origen fúndico o de la unión gastroesofá- incluso en ocasiones falta absoluta de movimiento o aperistalsis. Fun-
gica, aún hubiera sido más correcta la respuesta. damentalmente la esclerodermia se diferencia de la acalasia en la
2. Los situados en mediastino posterior: extensión de un microcítico manometría en que el esfínter esofágico inferior está hipotónico (por
de pulmón y los linfomas. eso la respuesta correcta es esclerodermia) y se relaja bien con la
deglución (ver tabla en página siguiente).
Pregunta 3.-R: 2 El síndrome de Sjögren puede dar lugar, en ocasiones, a disfagia
Entre las complicaciones de la acalasia cabe citar: alta por la intensa sequedad de la mucosa.
1. Disminución de peso por la desnutrición, debida a que por la El síndrome de Boerhaave es la perforación espontánea del esófa-
disfagia el paciente ingiere menor cantidad de alimento. go, generalmente tras una maniobra intempestiva, como un vómito o
2. La esofagitis que se produce por el contacto repetido del alimento un valsalva. Presenta la clínica de dolor retroesternal, semejante a la
con la pared esofágica. En ocasiones se debe a una infección perforación iatrógena y, además, derrame pleural con líquido rico
añadida por Cándida; pero no es posible que sea por reflujo, ya en amilasa.
que en esta enfermedad el esfínter está hipertónico en reposo y
Comentarios TEST

no permite el reflujo gástrico al esófago. Pregunta 7.-R: 3


Es de interés destacar que: El antecedente de pirosis y regurgitación es altamente sugestivo de
• Los pacientes tratados con dilatación endoscópica tienen pos- enfermedad por reflujo gastroesofágico ácido. Con estos síntomas no
teriormente reflujo al quedar el esfínter incompetente. se precisan pruebas diagnósticas y la actitud adecuada es instaurar
• Si un paciente con clínica de pirosis por reflujo gastroesofágico tratamiento con inhibidores de la bomba de protones (no con anti-
presenta de forma simultánea desaparición de su pirosis (es H2) y asumir el diagnóstico si desaparece la sintomatología. La apari-
decir, mejoría del reflujo) y a la vez aparición de disfagia se ción de dolor o disfagia plantea que ha surgido una complicación:
debe sospechar que se ha establecido una acalasia. bien una estenosis péptica o un carcinoma, por lo que es necesario
3. Tienen más riesgo de cáncer de esófago. la realización de una endoscopia (la respuesta correcta). La pH-metría,
4. El déficit de vitamina B12, respuesta correcta, no se puede atribuir prueba de certeza de la enfermedad por reflujo gastroesofágico, tie-
a la acalasia, ya que los problemas para su absorción no se deben ne sus indicaciones:
a patología esofágica y sí a gástrica (gastritis autoinmune), insufi- 1. Refractariedad al tratamiento médico.
ciencia exocrina pancreática, sobrecrecimiento bacteriano o al- 2. Valoración pre y postquirúrgica.
teraciones en la pared del ileon distal. 3. Síntomas predominantes extraesofágicos, por ejemplo. tos noc-
turna, asma o laringitis.
Pregunta 4.-R: 1
El tratamiento de elección de la acalasia es la dilatación neumáti- El test de Bernstein, o de provocación con una ligera cantidad de
ca endoscópica, por ser menos cruenta que la cirugía; pero cuatro ácido, se utiliza para aquellos casos en los que predomina una clí-
circunstancias obligan a plantear como primera opción la cirugía: nica con episodios aislados y limitados de intenso dolor retroesternal
1. El fracaso de las dilataciones endoscópicas (respuesta correcta la 1). parecido al angor y con pH-metría no diagnóstica.
2. La imposibilidad de someterse a la dilatación por presentar ero- El esofagograma con bario tiene poca utilidad, únicamente como
siones o divertículos en el esófago distal, ya que existiría riesgo de prueba complementaria a la endoscopia en el estudio de las estenosis.
hemorragia o perforación.
3. Cuando deseamos plantear al paciente un tratamiento definitivo. Pregunta 8.-R: 4
La miotomía de Heller es más eficaz: presenta menos fracasos que El esófago de Barrett es una complicación del reflujo. Se considera a
las dilataciones. Por eso en adolescentes y jóvenes se prefiere de su vez una forma de peor pronóstico si se acompaña de displasia en
primera elección el tratamiento quirúrgico. Éste se recomienda alguna de las biopsias. Se considera displasia grave si la displasia aparece
por vía laparoscópica y con cirugía antirreflujo asociada. en todas las muestras y se comprueba por dos anatomopatólogos y se

1
DIGESTIVO
CTO Medicina 1ª Vuelta

Pregunta 6. Características clínicas de las principales enfermedades esofágicas que cursan con disfagia.

ESPASMO ESOFÁGICO
ACALASIA ESCLERODERMIA CARCINOMA
DIFUSO

A todas las edades.


Edad A todas las edades. 30 - 35 años. 60 años.
Media 40 años.
Sexo Mujer = Hombre. Mujer > Hombre. Hombre > Mujer.
Generalmente no, tampoco si
Pirosis No. S í. No.
produce acalasia secundaria.
Cuando ya está avanzado e
Dolor No. No. Típico. indica generalmente
irresecabilidad.
Tanto para sólidos como
Progresiva, inicialmente para sólidos Disfagia tanto para sólidos para líquidos es
Características de Disfagia progresiva junto con
y existe pérdida de peso. Empeora como líquidos acompañada intermitente y no es
la disfagia pérdida de peso.
con el estrés y comidas rápidas. o no de reflujo. progresiva ni existe pérdida
de peso.

- De elección la manometría - Manometría: típicas


donde se observa presión del contracciones
- Endoscopia y biopsia.
EEI normal o aumentada. - En manometría la repetidas de gran
Es más frecuente en el
La presión basal del cuerpo presión del EEI basal amplitud y simultáneas
tercio medio,
Diagnóstico suele estar aumentada. está disminuida pero que comienzan en la
luego tercio inferior
Durante la deglución aparecen la relajación del parte inferior del
y luego superior.
contracciones simultáneas. esfínter es normal. esófago.
- Bario: ausencia de cámara aérea. - Bario: típica imagen en
Imagen en "pico de loro". sacacorchos.

asume que el riesgo de un adenocarcinoma injertado en el esófago de duce lesión histológica, por tanto no hay portadores, sólo enfermos,
Barret es muy elevado, por tanto se indica la esofaguectomía de la zona aunque frecuentemente asintomáticos.
afecta. A diferencia de una esofagitis por reflujo no complicada, el esó- En relación con el diagnóstico de la infección debemos recordar
fago de Barrett requiere controles endoscópicos para valorar el riesgo que (ver tabla):
de progresión a adenocarcinoma aunque el paciente se encuentre asin- 1. El cultivo de la mucosa antral es la prueba de certeza, pero es
tomático. Si se observaron displasias se recomienda endoscopia cada poco sensible y difícil de realizar.
seis meses. Si no hay displasia, como en el presente caso, cada año y 2. El estudio histológico de la mucosa con tinción de Giemsa es muy
medio o dos años. eficaz para el diagnóstico.
El Barrett está provocado por el reflujo, sin embargo, puede persis- 3. La prueba de la ureasa sobre histología o el test del aliento con
tir aunque mejore o cure la esofagitis. El paciente debe tomar ome- urea marcada con isótopo C13 son muy rentables. Esta última
prazol porque es muy probable que en el momento del diagnóstico tiene una sensibilidad y especificidad superior al 95%, siempre
tenga una esofagitis acompañante, al menos moderada. que el paciente no esté tomando antibiótico o inhibidor de la
La respuesta a señalar es la 4, pues es muy controvertida en el bomba de protones al realizarse la prueba.
esófago de Barret la relación entre adenocarcinoma de esófago y
colonización por H.pylori; actualmente se piensa que no guardan
relación. Pregunta 11. Fiabilidad de las pruebas diagnósticas del H. pylori.

Comentarios TEST
Pregunta 9.-R: 1 123456 7487959
9 6  626427967
El carcinoma epidermoide sigue siendo el grupo histológico más
frecuente. Desgraciadamente se diagnostica cuando aparece disfagia 12345672879
4 7 
76727 44

y significa que la enfermedad está muy avanzada y, por ello, la super-
vivencia a los 5 años oscila entre el 15 al 20%.
22222222
844  

2 72
72 2 7
En ocasiones se aprecian ulceraciones visibles en el estudio bari- !"68
3866
tado. La respuesta correcta es la 1, porque el adenocarcinoma supo-
ne aproximadamente un 10% o menos, aunque en los últimos años &7
2
2677
ha aumentado su incidencia, posiblemente en relación con el esófa- 22222222#8
$4 % 67
87
224
go de Barrett. 8
5
8
4
22222222'787
2 7 ( )68
Pregunta 10.-R: 3 222222228
$
 267
Para los tumores esofágicos malignos podemos concretar que para:
• El diagnóstico es de elección la toma de biopsias por endoscopia. *62464562
• Evaluar metástasis en pulmón, hígado u otras localizaciones: el 12&2
784 % 766

2876+8

TAC y la resonancia magnética son las mejores pruebas. ,-272 7+.
• Conocer si hay adenopatías tumorales, estadiaje N, es de elec- /42
8
727867
ción el TAC torácico.

07
28
$2 2 1
• Determinar el grado de extensión intramural en el esófago es la
ecografía endoscópica (respuesta correcta). 1237644 % 28
27278
4
7
7
4
41
Pregunta 11.-R: 5
328+
24277 
Es un gram negativo y flagelado (respuesta correcta la 5), que
7272&35361
afecta a la mucosa antral y, en ocasiones, a la duodenal próxima al
píloro; por tanto no se asocia siempre a colonización duodenal. Pregunta 12.-R: 5
Suele encontrarse en las capas profundas del moco o entre éste y el La gastrina aumenta, en general, como consecuencia de la re-
epitelio, pero no invade la lámina propia. Su infección siempre pro- ducción de la acidez gástrica, por tanto la hipoclorhidria es la

2
DIGESTIVO
1ª Vuelta CTO Medicina
causa más frecuente (respuesta correcta la 5). Por tanto las situacio-
nes que causan hipoclorhidria provocan aumento de la gastrina, Pregunta 17. Diferencias entre úlcera duodenal y gástrica.
como son las gastritis atróficas y el uso de anti-H2 o inhibidores de
la bomba de protones. No lo causan el sucralfato, que es un protec- 123456789
84 62 1234567 536
tor de la superficie ulcerada o erosionada pero no un antisecretor.
Es cierto que algunos carcinomas gástricos se asocian a hipoclor- 8
6 27
1234256789
4 6
hidria y, por tanto, a aumento de gastrina. Esta hipoclorhidria de los 9
4 68
tumores se debe a que coexiste una gastritis atrófica severa, no es

56 8 262 2
6
8492
6
por tanto un efecto tumoral. Otras causas de hipergastrinemia son 266
6
22

los gastrinomas y menos frecuentemente los tumores carcinoides.


89

Pregunta 13.-R: 5

36263 4 2 6
4
64
682

La profilaxis de la úlcera por AINES, o gastroprotección, se realiza


con omeprazol o misoprostol. No se ha demostrado utilidad de los 868  865  !865 
anti-H2, sucralfato o antiácidos. Aunque el riesgo de padecer una le-
sión gástrica, incluso una úlcera complicada, es superior por el mero
72
5 " # $ % #
hecho de tomar AINES, se considera que sólo procede administrar 43543 7 38
&
68 8↑ &
68 8↓
omeprazol o misoprostol cuando coexisten factores de riesgo que au- 662
mentan éste de forma intolerable. Son factores de riesgo: edad supe-
' 
829(
 ' 
829(

rior a 70 años, antecedente de úlcera, estar anticoagulado, altas dosis
de AINES, combinaciones de los mismos, uso concomitante de corti-
2 36 )9 2 8)9
coides y una enfermedad grave (no necesariamente estar hospitaliza-
do). En el presente caso la paciente no tiene ningún factor de riesgo, 12 2
6 7
por lo que la respuesta adecuada es no recomendar ninguna medida *2

66 -2

668./ #07
farmacológica. 
2363
4 
967 92

6 89 
92

6 7
242 2,
Pregunta 14.-R: 1 +
4 ,
Los anticuerpos anticélula parietal gástrica se usan para el diagnós-
tico de la gastritis autoinmune (tipo A). Aunque son muy sensibles, son 128 
627
3   96829
27
2   968.
poco específicos y pueden aparecer títulos, en general moderados, en
otras enfermedades autoinmunes o en sujetos sanos. Los antifactor in-
6 3
9
26078 8
.26
6

6 60
trínseco son más específicos; sin embargo para el diagnóstico es obli- 46
gado una biopsia del fundus gástrico, donde se objetive la atrofia. La
anemia macrocítica sugiere déficit de B12, por todo ello la respuesta 438!6 %"#82 8/865  4
242 2
correcta es la primera.
Un estudio baritado no permite hacer el diagnóstico de gastritis, ni Pregunta 18.-R: 3
aguda ni crónica. Cuando una úlcera gastroduodenal presenta una hemorragia di-
gestiva y se confirma la infección por H.pylori, la erradicación del
Pregunta 15.-R: 4 microorganismo es una prioridad. Se inicia el tratamiento oral en
Entre las causas de pliegues gástricos engrosados cabe destacar: cuanto el paciente comienza a tener una ingesta de alimentos. Se usa
1. Sífilis. la triple terapia con omeprazol, amoxicilina y claritromicina, cuya
2. Tuberculosis. eficacia es del 95%; pero en caso de alergia o intolerancia a alguno
3. Sarcoidosis. de los antibióticos se usa metronidazol y timidazol, no bismuto coloi-
4. Linfoma. dal, por eso la respuesta correcta es la 3. Éste se utiliza en combina-
5. Carcinoma gástrico, sobre todo tipo difuso. ción con tetraciclina, metronidazol y omeprazol en caso de que la
6. La enfermedad de Ménètrier. La asociación de engrosamiento en triple terapia previa no consiga la erradicación de H.pylori. Si, aún
fundus, dispepsia e hipoalbuminemia, sin que se observen reac- con esta triple terapia, no se consiguiera erradicar, se deberían tomar
tantes bioquímicos de enfermedad aguda, sugiere este diagnósti- cultivos de la mucosa para obtener antibiograma y así seleccionar la
co. La confirmación sería histológica. terapia.
Comentarios TEST

Después de una úlcera con hemorragia es obligado confirmar la


Pregunta 16.-R: 3 erradicación de H.pylori con un test del aliento. En el momento del
El mecanismo por el cual los AINES producen gastrotoxicidad es sangrado de una úlcera se deben evitar las biopsias de los bordes,
por la reducción de las prostaglandinas gástricas como consecuencia pero es que además en esta pregunta se trata de una úlcera duode-
de la inhibición de la COX-1. A más poder de inhibición de la COX-1 nal, por lo que de todas formas es innecesaria la biopsia.
más gastroerosivo es el fármaco. En la escala de gastroerosividad son
piroxicam y ketorolaco los de mayor capacidad erosiva e ibuprofeno Pregunta 19.-R: 1
el de menor. Entre los nuevos AINES inhibidores selectivos de la COX- En las úlceras gástricas debe confirmarse siempre la erradicación
2 se encuentran los menos gastroerosivos de todos: rofecoxib (res- de H.pylori tras completar tratamiento erradicador siendo altamente
puesta correcta) y celecoxib. Éstos reducen un 50% el riesgo recomendable en las duodenales (por tanto no deben realizarse siem-
gastroerosivo frente a fármacos tradicionales como diclofenaco o na- pre de forma obligada, aunque ésto no es contradictorio con el
proxeno. concepto de altamente recomendable), pasando a ser obligatorio si
las duodenales se complicaran con hemorragia.
Pregunta 17.-R: 4 La prueba de elección es el test del aliento y para evitar falsos
La causa más frecuente de úlcera es la infección por Helicobacter negativos debe realizarse entre 15 días y un mes después de haber
pylori. Sus úlceras aparecen fundamentalmente en antro gástrico y, suspendido omeprazol y antibióticos.
sobre todo, en la primera porción del duodeno.
Las úlceras gástricas son más grandes que las duodenales, por eso Pregunta 20.-R: 5
tienen más riesgo relativo de sangrado digestivo. Aunque son más gran- Las dos pruebas más utilizadas para el diagnóstico de la infección
des, las gástricas se perforan menos, dado el mayor espesor de la pa- por H.pylori en ulcus gástrico son el test de la ureasa y el estudio con
red gástrica. Se calcula que entre el 6 y el 8% de las úlceras gástricas tinciones de la mucosa biopsiada. Su sensibilidad depende de la
calidad de la muestra, por eso se debe evitar tomar muestras de las
pueden ser malignas. Cuando una úlcera aparece en fundus, la prime-
zonas con mayor atrofia o metaplasia, pues en estas localizaciones es
ra posibilidad es que siga siendo benigna, pero respecto a lo que suce-
muy difícil demostrar la presencia de la bacteria.
de en otras regiones gástricas aumentan las probabilidades de que sea
El tratamiento erradicador con la triple terapia requiere entre 7 y
maligna. Las diferencias entre duodenales y gástricas se observan en
10 días y, en principio, no es necesario llegar hasta 14 días, pues el
la tabla. beneficio adicional es mínimo. Las úlceras gástricas una vez concluido
el tratamiento erradicador requieren un mantenimiento durante un

3
DIGESTIVO
CTO Medicina 1ª Vuelta
período no concretado de semanas (oscila entre 4 y 8) con un fármaco 1. Incumplimiento dietético, que es la respuesta correcta.
antisecretor, preferentemente un anti-H2 (esta es la respuesta correcta) 2. Linfoma tipo T intestinal.
y no es necesario en las duodenales, a no ser que debutaran sangrando 3. Esprue colágeno.
o fueran gigantes. La cirugía del ulcus está fundamentalmente motivada 4. Otros diagnósticos distintos a la propia enfemedad celiaca, como
por sus complicaciones, no por su localización. En un ulcus no refracta- el esprue autoinmune o el tropical.
rio y sin otros síntomas más que los habituales de la enfermedad ulcero-
sa, no es necesario solicitar gastrina. Pregunta 27.-R: 4
En general secretora y acuosa suele ser un término equivalente.
Pregunta 21.-R: 2 Son causas los tumores endocrinos, como el vipoma, pepoma, o
El paciente presenta un factor de riesgo para la gastropatía y úlce- tumor carcinoide (respuesta correcta), que liberan sustancias estimu-
ra complicada por AINES, como es la edad mayor de 70 años, por lo ladoras de la liberación de líquido a la luz intestinal o que alteran la
que requiere omeprazol oral (la respuesta correcta). permeabilidad del epitelio intestinal. Otras causas son los pólipos
Parece que erradicar H.pylori reduciría el riesgo de úlcera com- vellosos grandes y múltiples. En los casos agudos, las diarreas por
plicada al tomar un AINE, pero no se ha demostrado que evite tomar toxinas bacterianas, como Staph.aureus o E.Coli.
el gastroprotector. En relación a la ranitidina bismuto coloidal es un La vagotomía y el intestino irritable son diarreas motoras y el hi-
fármaco con actividad frente a H.pylori y buena alternativa al ome- dróxido de magnesio y la lactulosa osmóticas.
prazol en la triple terapia.
Pregunta 28.-R: 5
Pregunta 22.-R: 5 La enteropatía pierdeproteínas no es una enfermedad, sino una
No tiene factores de riesgo para asociar gastroprotector al consu- modalidad de expresión clínica en la que predominan la pérdida de
mo de ibuprofeno, por lo que no necesita misoprostol y no es nece- proteínas por la luz intestinal frente a la maldigestión o malabsorción
sario sustituir ibuprofeno por rofecoxib. proteica de otros procesos. Son causa la propia enfermedad celíaca,
El tabaquismo tiene un efecto deletéreo comprobado básicamente las infiltraciones tumorales de la pared intestinal, la linfangiectasia o
como elemento que dificulta la cicatrización de las úlceras. La dieta las obstrucciones al drenaje del conducto torácico, como el carci-
no tiene influencia. noma de esófago o la pericarditis constrictiva. La prueba diagnóstica
Una dispepsia inducida por antiinflamatorios debe tratarse reti- utilizada con más frecuencia es el aclaramiento entre las cifras fecales
rando si es posible el antiinflamatorio y añadiendo un antiácico o y séricas de alfa1-antitripsina.
un anti-H2 durante un tiempo limitado, por eso la respuesta co- El hipertiroidismo provoca una diarrea por hipermotilidad (res-
rrecta es la 5. puesta correcta) y no causa una enteropatía pierdeproteínas.

Pregunta 23.-R: 5 Pregunta 29.-R: 5


H.pylori es un reconocido agente carcinógeno debido a la atro- En general siempre existe un factor o antecedente personal aso-
fia gástrica y metaplasia que induce. La gastritis tipo A, frecuente- ciado que justifica el sobrecrecimiento bacteriano. En los gastrecto-
mente asociada a anemia perniciosa, es también un factor de riesgo mizados es muy frecuente (respuesta correcta) sobre todo tipo Billroth
para el cáncer. El adenocarcinoma en el Barrett puede, a veces, II, también en los by-pass intestinales, síndrome de intestino corto,
localizarse dentro de lo que se considera anatómicamente el estó- divertículos yeyunales, fístulas y asas ciegas que son los ejemplos más
mago. El Lynch tipo II, asocia cáncer colorrectal hereditario sin frecuentes.
pólipos con cáncer gástrico. El alcoholismo crónico puede facilitar Produce maldigestión por alteración de las sales biliares. A veces
una gastritis, pero no hay evidencia de que sea un factor causal de hay ligera atrofia intestinal, que no es responsable de los trastornos
úlcera o cáncer gástrico. nutricionales. Si no se resuelve la enfermedad subyacente, es fre-
cuente la recurrencia. La prueba diagnóstica de certeza es el recuen-
Pregunta 24.-R: 2 to de aerobios y anaerobios en jugo intestinal aspirado superior a
Habitualmente en una diarrea aguda, de menos de 14 días de 100.000 UFC; pero la dificultad de esta prueba obliga a utilizar los
evolución, no se requiere obtener el diagnóstico etiológico, pues en test del aliento, siendo el más rentable el de xilosa con carbono
general son procesos leves y autolimitados. En caso de deshidrata- isotópico.
ción, sepsis, estado de inmunosupresión o edad muy avanzada, al
igual que en las duraciones prolongadas del cuadro diarreico de más Pregunta 30.-R: 5
de 10 días es necesario obtener el diagnóstico. La primera prueba, y Esta pregunta hace referencia al diagnóstico diferencial de la asci-
por eso es la respuesta correcta, es el estudio de los leucocitos fecales. tis quilosa:
Si es positivo sugiere un origen inflamatorio y hay que esperar al 1. Los traumatismos abdominales con fistulas linfático-peritoneales.

Comentarios TEST
resultado del coprocultivo para el diagnóstico. En caso de que las 2. Los linfomas.
pruebas bacteriológicas no consiguieran el diagnóstico, se realizaría 3. La cirrosis hepática ocasionalmente.
una colonoscopia. Si no hay leucocitos en las heces, se entiende que 4. La linfangiectasia primaria (respuesta correcta). Se acompaña de
es una diarrea acuosa aguda, en general debida a toxinas bacteria- edemas en MMII, linfopenia y aclaramiento de alfa-1-antitripsina
nas, que suelen ser autolimitadas y precisan diagnósticos microbio- elevado pues es una enteropatía pierde proteínas.
lógicos y epidemiológicos específicos.
Pregunta 31.-R: 1
Pregunta 25.-R: 2 La causa más frecuente de hemorragia digestiva baja en un ancia-
Para el diagnóstico de enfermedad celíaca se precisa una biopsia no es el sangrado por divertículos de colon, sobre todo los situados
que demuestre atrofia de las vellosidades, hiperplasia de las criptas e en el hemicolon derecho. Sin embargo, suele ser autolimitado y
infiltración linfoplasmocitaria de la lámina propia. Si, además, el raramente recurrente.
estudio inmunológico es positivo, la actitud diagnóstica pasa por La angiodisplasia aparece como segunda causa, pasando a ser la
retirar el gluten de la dieta, observar la mejoría clínica y la negativiza- primera en caso de sangrado recurrente. Se observa con más fre-
ción de los anticuerpos. Si ésto ocurriera no es obligada una segunda cuencia si coexiste estenosis aórtica o insuficiencia renal y para su
biopsia dos años después de la primera que confirme la recupera- valoración la colonoscopia no debe limitarse hasta ángulo esplénico
ción histológica. como en este caso, sino realizarse completa hasta ciego. Esta prueba
Los linfocitos intraepiteliales están aumentados en número en es la de mayor rentabilidad diagnóstica superior incluso a la propia
esta enfermedad y desempeñan un papel patógeno importante. Se arteriografía permitiendo el tratamiento con fotocoagulación en los
considera celíaca latente a la positividad de los anticuerpos y au- casos en que el sangrado se limite a un número reducido de lesiones.
mento de linfocitos intraepiteliales en un familiar de un celiaco. La
asociación al HLA es fundamentalmente al DQ2 y no al B51. En Pregunta 32.-R: 1
relación con los anticuerpos se utilizan los antigliadina IgG e IgA, los Las causas más frecuentes de hemorragia digestiva baja varían
antiendomisio IgA y, recientemente, los antitransglutaminasa IgA, que según la edad. En los mayores de 60 años, que es el grupo con
son muy sencillos de realizar, sensibles y específicos. mayor incidencia, el orden por frecuencia es:
1. Divertículos.
Pregunta 26.-R: 2 2. Angiodisplasia.
Entre las causas de refractariedad al tratamiento sin gluten se en- 3. Pólipos.
cuentran por orden de probabilidad: 4. Neoplasia.

4
DIGESTIVO
1ª Vuelta CTO Medicina
Teóricamente la prueba de elección es una colonoscopia completa, Pregunta 35.-R: 1
pero esta prueba requiere una preparación. Una sigmoidoscopia se Los ANCA patrón perinuclear aparecen en el 70% de las colitis
puede realizar casi de forma inmediata por lo que puede ser la prime- ulcerosas, pero es también conveniente que recordemos que en el
ra prueba a realizar sobretodo si el sangrado es relevante. El algoritmo MIR aparecen en la poliangeítis microscópica, hepatitis autoinmune,
diagnóstico se observa en la figura. La gastroduodenoscopia no aporta colangitis esclerosante, y el patrón citoplasmático en la enfermedad
nada de interés en una HDB y la arteriografía es útil sólo si el sangrado de Wegener.
es cuantioso. Actualmente se han descrito en algunos casos de Crohn anticuer-
pos antisacharomyces (ASCA).
Son características de la colitis ulcerosa:
1. Es muy infrecuente en fumadores.
2. Afecta siempre al recto y sólo en un 20% a todo el colon.
3. El síntoma clínico más frecuente es la rectorragia, pero casi siem-
pre con gran emisión de moco, incluso en las remisiones.
4. Aproximadamente entre el 6 y el 10% de los que presentan colitis
ulcerosa, y no el 50%, asocian una colangitis esclerosante.

Pregunta 36.-R: 4
La descripción endoscópica de una mucosa eritematosa y granu-
lar en colon o recto es muy sugestiva de colitis ulcerosa, más aún si se
describieran lesiones petequiales, erosiones o úlceras. No obstante,
el diagnóstico definitivo no es endoscópico sino que se debe dispo-
ner de histología compatible aunque no sea patognomónica.
La presencia de sepsis, deshidratación severa, o, como en este
caso, seis ó más deposiciones al día, supone que el episodio es severo
y, por tanto, el tratamiento es con 5-aminosalicilatos más esteroides
(respuesta nº 4). Los salicilatos como único tratamiento se emplean
en las formas leves. En la colitis ulcerosa el metronidazol sólo se
utiliza si se asocia megacolon tóxico.

Pregunta 37.-R: 4
En general, tanto la colitis ulcerosa como la enfermedad de Crohn
pueden presentar los mismos síntomas, pero son más típicas de colitis
ulcerosa la rectorragia y la friabilidad al estudio endoscópico (res-
puesta correcta) y más propio de la enfermedad de Crohn: dolor
abdominal sobre todo en fosa ilíaca derecha, pérdida de peso, fie-
bre, presencia de masa abdominal, fístulas y cuadros obstructivos o
pseudoobstructivos. Es conveniente recordar que aunque la enfer-
Pregunta 32. Hemorragia digestiva baja. medad de Crohn pueda afectar a capas profundas de la pared intes-
tinal, es más frecuente la presencia de perforaciones en la colitis
Pregunta 33.-R: 1 ulcerosa, posiblemente por su asociación a megacolon.
La presencia de melenas es indicativo de una hemorragia digesti-
va, en el 80% de los casos alta. La causa más frecuente de hemorragia Pregunta 38.-R: 1
digestiva en un paciente previamente sano es el ulcus duodenal, Los esteroides son útiles para lograr la remisión. Pero tanto en la
(respuesta correcta) en segundo lugar el ulcus gástrico, en tercero las colitis ulcerosa como en la enfermedad de Crohn, su uso como
erosiones y gastritis de stress. En este caso coexiste anemia ferropéni- mantenimiento no garantiza una remisión prolongada (respuesta
ca que en una mujer joven es secundaria al propio ulcus o a pérdi- correcta la número 1). Cuando se suspenden o se reduce la dosis no
das menstruales. Es de interés recordar que en un anciano la anemia se modifica el riesgo de padecer un nuevo brote. Por tanto, en el
ferropénica obliga a excluir la neoplasia de colon. mantenimiento de la enfermedad de Crohn se opta ante un primer
brote leve o moderado por no tomar ninguna medida, salvo si la
Pregunta 34.-R: 3 afectación en el brote fue una colitis que mejoró con salicilatos, en
Comentarios TEST

La presencia de aftas en colon sugiere una colitis de Crohn y en cuyo caso se puede optar por probar a continuar con este tratamien-
este caso leve-moderada. Su tratamiento incluye: to como mantenimiento. En caso de brote inicial muy grave o brotes
1. Por afectar el colon se recomienda, aunque no es obligado, me- repetidos, se pasa directamente al mantenimiento con un inmuno-
tronidazol a dosis no muy elevadas como inmunomodulador. supresor como la azatioprina.
2. 5-aminosalicilatos, p.e. mesalazina, aunque su eficacia es menor En los brotes graves refractarios a dosis altas de corticoides puede
que en la colitis ulcerosa y frecuentemente se necesitan añadir emplearse ciclosporina i.v.
dosis bajas de esteroides orales o en enema. En algunos casos los enfermos de Crohn tienen diarrea no por
actividad de la enfermedad sino por sus secuelas:
Si hubiera sido severa los corticoides se pautan desde el inicio y a 1. Sobrecrecimiento bacteriano.
dosis altas (ver figura). 2. Intestino corto postquirúrgico.
3. Ante extensa afectación ileal pueden no absorberse bien los áci-
dos biliares que al pasar a colon provocarían una diarrea secreto-
ra cuyo tratamiento es la colestiramina.

Pregunta 39.-R: 4
La presencia de diarrea con múltiples deposiciones y deshidrata-
ción en una enferma con colitis ulcerosa sugiere un brote severo. La
distensión abdominal no es propia del brote y sugiere un megacolon
tóxico, que se diagnostica en una Rx simple de abdomen con diáme-
tro luminal de colon transverso superior a 6 cm. Esta situación es muy
grave y obliga al tratamiento propio de las formas severas más metro-
nidazol. Si no responde en 48 h o se perfora, está indicada la colec-
tomía urgente.

Pregunta 40.-R: 5
Es una lesión por alteración de la microvasculatura arterial y, por
Pregunta 34. Tratamiento de un brote de enfermedad inflamatoria intestinal. tanto, la arteriografía no es diagnóstica y sí lo es la histología del colon;
por esto, tanto las respuestas 1 como 3 son falsas. Afecta al colon, sobre

5
DIGESTIVO
CTO Medicina 1ª Vuelta
todo al ángulo esplénico, pero respeta al recto. Las imágenes en la Rx en tamaño, más riesgo de neoplasia. El tipo histológico velloso de los
huella dactilar se observan también en las formas agudas. adenomas tiene más riesgo que el tubular. El hiperplásico no es una
Los síntomas, generalmente diarrea, suelen remitir de forma habi- lesión premaligna. El área de intestino grueso que esté afectada no
tual en unas pocas semanas y no suele haber recurrencias, por eso la se asocia a un mayor riesgo de transformación neoplásica (respues-
respuesta correcta es la 5. ta correcta la 4).

Pregunta 41.-R: 1 Pregunta 44.-R: 3


Para el diagnóstico de la enfermedad diverticular del sigma o co- Los pólipos, tanto hiperplásicos como adenomatosos, suelen ser
lon la prueba de elección es el enema opaco. Está contraindicado su asintomáticos; pero en caso de producir síntomas suelen presentar
práctica si se sospecha una complicación como la diverticulitis, por rectorragia o hematoquecia. Unicamente producen diarrea cuando
el riesgo de perforación y peritonitis clínica. El diagnóstico de diver- son de tipo velloso y además muy indiferenciados, múltiples y de
ticulitis es clínico y bioquímico: anciano con fiebre, dolor en fosa gran tamaño. La intususcepción es menos frecuente y puede provo-
ilíaca izquierda y leucocitosis. La sospecha de una diverticulitis com- car dolor e íleo.
plicada con un absceso de pared por persistencia de la fiebre o una
palpación muy sugestiva de peritonismo, se comprueba mediante Pregunta 45.-R: 2
un TAC. Esta prueba es la más rentable para el diagnóstico de masas, En población sana con edad superior a los 50 años y sin factores
tumores y abscesos del espesor de la pared intestinal. de riesgo asociados se recomienda un screening con un test de he-
morragias ocultas cada dos años. Pero siempre que exista un factor
Pregunta 42.-R: 3 de riesgo personal (antecedente de colitis ulcerosa o como en este
La enfermedad diverticular del colon es muy frecuente, general- caso pólipos adenomatosos previos) o familiar por antecedente de
mente es asintomática. En caso de producirse clínica es más frecuen- neoplasia de colon, el cribado debe hacerse con colonoscopia cada
te la diverticulitis que la hemorragia; pero es tan prevalente la enfer- año y medio o dos. Este plan es indefinido, pues es la única forma de
medad que la hemorragia diverticular, sobre todo localizada en co- detectar precozmente un cáncer de colon.
lon derecho, es la causa más frecuente de hemorragia digestiva baja
en mayores de 60 años. Como no suele ocurrir sobre un divertículo Pregunta 46.-R: 2
inflamado: es indolora, autolimitada y no recidiva; por eso la falsa es Las características más relevantes de las poliposis familiares se
la número 3. ilustran en la figura. Son factores de riesgo bien reconocidos para
el cáncer de colon: la presencia de cualquier poliposis familiar,
Pregunta 43.-R: 4 excepto en el síndrome de Cowden; los antecedentes familiares
El punto fundamental que da mal pronóstico en cuanto al po- de cáncer de colon y los antecedentes de enfermedades como la
tencial de malignización de un pólipo es la presencia y grado de colitis ulcerosa o el Crohn. No así el síndrome del intestino irrita-
displasia en su interior. Lógicamente a más pólipos y de mayor ble (por eso es la respuesta correcta). Otro tema bien distinto es

Comentarios TEST

Pregunta 46. Características más relevantes de las poliposis familiares.

6
DIGESTIVO
1ª Vuelta CTO Medicina
que en ocasiones su cuadro clínico puede ser parecido al de un enfermedad y por tanto no tiene sentido someterle a colonoscopias
cáncer de colon incipiente y, por eso, cuando la clínica de colon de seguimiento (respuesta correcta es la número 4).
irritable aparece por primera vez en sujetos de más de 50 años se
debe excluir la presencia de un cáncer de colon mediante la rea- Pregunta 52.-R: 5
lización de pruebas diagnósticas como la colonoscopia o el ene- El aumento aislado de bilirrubina indirecta, o no conjugada, es
ma opaco. generalmente leve o moderado, excepto en el recién nacido con
ictericia fisiológica que puede llegar a producir Kernicterus. Cau-
Pregunta 47.-R: 4 sas habituales son las anemias hemolíticas y el síndrome de Gilbert.
La colitis ulcerosa y la enfermedad de Crohn son enfermedades En ambos casos la bilirrubina no suele exceder de 4-5 mg/dl. Tam-
en las que aumenta el riesgo de padecer cáncer de colon. Este au- bién se observa en una forma o variante del Gilbert, como es el
mento es más notorio en la colitis ulcerosa. Crigler-Najar, donde alcanza cifras más importantes y puede ser
El riesgo depende fundamentalmente del grado de actividad de la incluso mortal en niños con el tipo I. Otras causas son el hipotiroi-
enfermedad y de la duración de la misma. Aumenta a partir de los 10 dismo, la lactancia materna o los estados de hipoalbuminemia
años en enfermos con formas crónicamente activas o con multibrotes; severa. Todas estas entidades no presentan bilirrubinuria. El sín-
por eso si la enfermedad debuta en edad juvenil hay un mayor drome de Dubin-Johnson se caracteriza porque la bilirrubina ya
potencial de padecer más brotes a lo largo de la vida y por tanto más conjugada no puede pasar a los canalículos biliares y, por tanto, la
riesgo de desarrollar un carcinoma. bilirrubinemia es de tipo conjugado o directo, por eso es la res-
Otro factor adicional es la extensión de la enfermedad, por ejem- puesta correcta. Muy parecido a este cuadro es el síndrome de
plo: no hay más riesgo de cancer cuando únicamente presentó Rotor, por tanto también con elevación predominante de la bili-
proctitis. rrubina conjugada. Estos dos últimos diagnósticos sí se acompa-
En los casos en que su severidad así lo indique y a partir de los ñan de bilirrubinuria.
10 años de evolución, se recomiendan colonoscopias cada dos
años para tomar biopsias y valorar la presencia de neoplasias pre- Pregunta 53.-R: 5
coces. Si se demuestran zonas displásicas se asume tal riesgo de Siempre que hay colestasis, un Dubin-Johnson o un síndrome de
neoplasia y se recomienda la colectomía; pero ésta no se indica Rotor, predomina la bilirrubina conjugada y se encuentra bilirrubina
por el mero hecho de llevar 15 años de evolución (por eso la en orina, puesto que ésta sí puede filtrarse por el riñón. El síndrome
respuesta incorrecta es la 4). de Gilbert (respuesta correcta) causa aumento de bilirrubina indirec-
ta y, por tanto, no cursa con hiperbilirrubinuria.
Pregunta 48.-R: 1
La causa más frecuente de cancer de colon hereditario, a edad Pregunta 54.-R: 4
inferior a los 40 años, es el carcinoma colorrectal hereditario no En un síndrome colestático la ecografía (respuesta correcta) es la
asociado a pólipos (respuesta correcta la número 1). Esta entidad es primera prueba que debe realizarse. Si se demuestra dilatación de la
cinco veces más frecuente que la poliposis adenomatosa familiar del vía biliar principal supone probablemente que se trate de una coles-
colon. tasis extrahepática y el siguiente paso diagnóstico es un TAC. En caso
de no obtener el diagnóstico, se realiza una colangiografía retrógrada
Pregunta 49.-R: 1 endoscópica. Esta técnica es cruenta: puede producir un 5% de
Un paciente con cáncer de colon necesita para su seguimiento: pancreatitis y un 1% de colangitis, por eso actualmente están cobran-
1. Colonoscopias periódicas. do importancia otras técnicas:
2. Ecografía abdominal para observar posibles lesiones metastásicas 1. La ecografía endoscópica de la vía biliar, situando el endoscopio
hepáticas. en duodeno se realiza un barrido de páncreas y vía biliar. Tiene
3. La determinación del antígeno carcinoembrionario (CEA). Su ele- una alta rentabilidad para las enfermedades del colédoco, sobre
vación significa recidiva tumoral (respuesta 1). El test de hemorra- todo la litiasis.
gias ocultas no está indicado por su baja rentabilidad para el 2. La colangiografía por resonancia magnética, que es incruenta y
seguimiento de las neoplasias de colon. no precisa contraste.
3. La colangiografía i.v. y la oral no tienen utilidad, salvo en los
Las segundas neoplasias pueden surgir en otra localización en casos previos a la litotricia biliar para evaluar si la vesícula es
un enfermo con cáncer de colon después de la cirugía. Por ejem- funcionante.
plo en un enfermo con colectomía puede aparecer una segunda
neoplasia en recto; pero esta situación, además de infrecuente, no Pregunta 55.-R: 5
tiene por qué cursar necesariamente con CEA elevado, puesto que La intoxicación por paracetamol produce un fallo hepático agu-
Comentarios TEST

este segundo tumor puede encontrarse en un estadío poco avan- do por un efecto tóxico directo por consumir los niveles de glutatión
zado. del hepatocito. Se produce con dosis superiores a los 8 gramos al
día, excepto en alcohólicos, en los que incluso se puede producir
Pregunta 50.-R: 5 con dosis dentro del rango terapéutico habitual. Tiene buen pro-
El que los anticuerpos antigliadina sean negativos hace muy im- nóstico si se consigue administrar precozmente N-acetilcisteína i.v.
probable que se trate de una celíaca o su variante esprúe colágeno. Las transaminasas están muy elevadas y se acompaña de alteracio-
En el síndrome de intestino irritable, tanto la visión colonoscópica nes en la coagulación y si hay fracaso hepático establecido, de
como las biopsias son normales. El hipertiroidismo da una diarrea encefalopatía (por eso la 5 es la respuesta correcta).
motora.
La respuesta correcta es la colitis linfocítica. Esta entidad se asocia Pregunta 56.-R: 3
con cierta frecuencia a la celiaca. En general o no se aprecian altera- El tratamiento de la ascitis por descompensación hidrópica de un
ciones endoscópicas en el colon o son leves e inespecíficas. El diag- cirrótico es (ver figura de la página siguiente):
nóstico se establece al obtener muestras por biopsia para estudio 1. En los casos leves-moderados: restricción dietética de sal y espiro-
histologico (hay que resaltar que en un protocolo de diarrea se to- nolactona con dosis progresivas según resultado si no fuera sufi-
man biopsias incluso con un aspecto endoscópico normal). Se apre- ciente la de 100 mg/día. En ocasiones puede ser necesario añadir
cia un infiltrado linfoplasmocitario y en ocasiones una banda de un segundo diurético.
colágeno en cuyo caso recibe el nombre de colitis colágena. El tipo 2. En los casos severos se puede optar por la combinación de espiro-
de diarrea que produce es secretora. nolactona y otro diurético como furosemida o torasemida, o bien
por la paracentesis evacuadora con reposición de albúmina i.v.
Pregunta 51.-R: 4 Ambas opciones son válidas, pero es más eficiente, pues ahorra
En un paciente de edad media la presencia de múltiples pólipos costes, la paracentesis.
adenomatosos y un cáncer en sigma o colon sugiere fuertemente 3. Si la ascitis es a tensión, la paracentesis evacuadora con reposi-
que se trate de una poliposis adenomatosa familiar del colon. Esta ción de albúmina es el tratamiento de elección (respuesta co-
entidad, dada su herencia autosómica dominante y alto grado de rrecta).
penetrancia, obliga a los familiares a someterse a un cribado del 4. En los casos refractarios, surge la indicación de transplante hepá-
cáncer de colon. Este debe iniciarse antes de los 20 años y si se tico. Se ha comprobado que en esta situación la colocación de
demuestran pólipos extirpar el colon. Si con edad superior a 40 años TIPS resuelve la ascitis refractaria en el 30% de los casos.
no se han demostrado pólipos, es que ese familiar no va a padecer la

7
DIGESTIVO
CTO Medicina 1ª Vuelta
Pregunta 60.-R: 2
Las indicaciones de la profilaxis antibiótica con quinolonas ora-
les en la peritonitis bacteriana espontánea de un cirrótico son:
1. Sin antecedentes de haber padecido antes una peritonitis: si se
presenta una hemorragia digestiva (respuesta correcta) o las prote-
ínas en líquido ascítico son inferiores a 1 gr/dl.
2. Como profilaxis secundaria después de un episodio de peritoni-
tis. Se mantiene la profilaxis con quinolonas de forma indefinida
hasta que el paciente deje de tener ascitis.

Pregunta 61.-R: 5
El síndrome hepatorrenal se produce como una forma de evolu-
ción terminal de una hepatopatía crónica avanzada. Pero también se
reconocen factores precipitantes del síndrome hepatorrenal: la peri-
tonitis bacteriana espontánea, la hepatitis alcohólica aguda y cada
vez con menor frecuencia el uso intempestivo de diuréticos.
Se trata de una vasoconstricción funcional irreversible, que afecta
básicamente a las arteriolas de filtración, pero en escasa cuantía a las
arterias de las cuales se obtiene la perfusión de la nefrona, por lo cual
ésta no resulta dañada desde el punto de vista morfológico y así se
entiende que exista un fallo en la filtración renal, y, por tanto, insufi-
ciencia renal progresiva y severa con un riñón histológicamente nor-
mal (respuesta correcta la 5).

Pregunta 62.-R: 5
El tratamiento de elección del sangrado por varices esofágicas es
la combinación de un tratamiento endoscópico como la esclerosis
o, si es posible, la ligadura endoscópica de las varices, más el farma-
cológico con somatostatina o un análogo de la vasopresina. Si este
tratamiento no es eficaz y continúa con sangrado, se indica la colo-
Pregunta 56. Manejo de la ascitis. cación de un TIPS, excepto en el estadío A de Child, en donde se
indica la cirugía urgente derivativa. Si esta recidiva es severa y no
Pregunta 57.-R: 3 permite esperar a estos procedimientos, hay que colocar una sonda
Para determinar la presencia de hipertensión portal se debe cons- de Sengstaken hasta que se pueda realizar el tratamiento intervencio-
tatar: nista.
1. Que las presiones están aumentadas en vena porta, determinadas Se deben transfundir hematíes si hay síndrome anémico, prescri-
por método cruento, o bir quinolonas orales para la profilaxis de la peritonitis y lactulosa
2. De forma indirecta, visualizando varices esofágicas o gástricas en para la profilaxis de la encefalopatía hepática.
la endoscopia. Está contraindicado el empleo de diuréticos o subir las dosis,
puesto que provocarían un mayor deterioro hemodinámico (res-
Otro tema bien distinto es evaluar si el origen de un líquido puesta correcta).
ascítico es atribuible o no a la hipertensión portal. Para ello se
utiliza el gradiente, o resta, entre la albúmina en suero y la albúmi- Pregunta 63.-R: 4
na en ascitis. Cuando esta resta da un resultado superior a 1,1 gr/dl La profilaxis primaria de la hemorragia digestiva por varices debe
se presume que la ascitis es por hipertensión portal, como sucede hacerse con propanolol o nadolol. Ha demostrado reducir el nú-
en cualquier tipo de cirrosis, Budd–Chiari, enfermedad venooclu- mero de primeros episodios de sangrado y la mortalidad atribuible
siva hepática, esquistosomiasis o trombosis de la porta. Si el gra- al mismo. Se debe iniciar siempre que se observen varices y que
diente es menor de 1,1 sugiere otros diagnósticos distintos al de éstas sean de riesgo, es decir, grandes (grado III ó IV). Si no se obser-
hipertensión portal, como la tuberculosis peritoneal (es la respues- van, como ocurre en este caso, o son pequeñas, únicamente se
ta correcta, ya que el gradiente es de 3,1) o la carcinomatosis peri- debe someter a vigilancia endoscópica periódica para ver cómo

Comentarios TEST
toneal. evolucionan las varices y por tanto no se precisa un tratamiento
farmacológico.
Pregunta 58.-R: 1
La presencia de células tumorales malignas en el líquido ascítico Pregunta 64.-R: 4
sucede porque el peritoneo está infiltrado por un tumor. En general La infección por el virus de la hepatitis C sucede por vía parenteral
suele ser metastásico fundamentalmente de colon o estómago (res- y por eso se observa en drogadictos, pero cada vez con mayor fre-
puesta correcta 1) y es independiente el que se asocie o no a metás- cuencia hay casos en que no se demuestra un antecedente que
tasis hepáticas. permita justificar la infección, debió ser, pues, inaparente. Por tanto
Esta situación confiere muy mal pronóstico, ya que la sobrevida es una respuesta falsa que la mayoría tengan historia de transfusión,
de una neoplasia con carcinomatosis peritoneal oscila, como me- esto sí ocurrió en los primeros años de descripción de la enferme-
diana, en los 6 meses. Si hubiera sido quilosa se nos describiría un dad, pero no en la actualidad.
aumento notable de los triglicéridos en el líquido. Se ha calculado que el 80% de los casos evolucionan a la croni-
cidad. El RNA es el mejor marcador para el diagnóstico. Inicialmente
Pregunta 59.-R: 2 se hace un cribado con los anticuerpos anti-VHC y, si son positivos, la
Se produce por la colonización y posterior infección del perito- infección se confirma solicitando amplificación genómica viral, que
neo por microorganismos propios de la flora intestinal del intestino demuestra si hay positividad para el RNA viral.
delgado: por eso se produce en primer lugar por gramnegativos,
como E.coli, y en segundo lugar por cocos gram positivos, incluido el Pregunta 65.-R: 3
neumococo. Para diagnosticar una hepatitis aguda se precisa, tanto en los casos
La clínica dolorosa es poco relevante. Es muy infrecuente el que tengan clínica como en los asintomáticos, demostrar un incre-
peritonismo, por eso la respuesta a señalar es la número 2. A dife- mento notable y agudo de las transaminasas. Una vez que tenemos el
rencia de las secundarias la presencia de anaerobios es excepcio- diagnóstico probable clínico-bioquímico o sólo bioquímico es cuan-
nal. El tratamiento es médico, con cefalosporinas de tercera gene- do se solicitan los marcadores virales y, en concreto, la serología IgM
ración. es la que va a ser positiva en los casos agudos (respuesta correcta).
La prueba diagnóstica de certeza es el cultivo del líquido ascítico, En la infección aguda por VHB los 5 marcadores de esta pregunta
pero, dada la gravedad de la enfermedad, se precisa iniciar un trata- son positivos, pero el que es específico de hepatitis aguda es el anti-
miento empírico y, para ello, es suficiente con demostrar un recuen- cuerpo IgM.
to de neutrófilos superior a 250/mm3 en líquido ascítico.

8
DIGESTIVO
1ª Vuelta CTO Medicina
Pregunta 66.-R: 1 Pregunta 69.-R: 3
La infección por el virus de la hepatitis C afecta también a órganos La positividad IgM para el virus A en una hepatitis aguda ofrece el
extrahepáticos, sobre todo a los ganglios linfáticos, generando riesgo diagnóstico de seguridad de hepatitis aguda por VHA. Por eso la
de linfomas y de enfermedades de naturaleza inmune como la apla- respuesta correcta es la número 3. En relación con la serología del
sia medular, eritroblastopenia, glomerulonefritis, liquen plano, sín- virus B, es probable que el enfermo se encuentre en una fase previa
drome seco, y, sobre todo la más frecuente, la crioglobulinemia mix- a la aparición de la respuesta inmune: todavía no la tiene porque es
ta esencial. Esta entidad es una vasculitis que cursa con crioglobuli- anti-S negativo. La asociación de: anti-E positivo, anti-Core positivo,
nas positivas en sangre, polineuropatía, afectación renal y púrpura. aunque no explican si es IgG o IgM, más un antígeno de superficie
La etiología es, prácticamente siempre, debida a la infección por el negativo sugiere que el paciente está atravesando el período ventana
virus de la hepatitis C. previo a la resolución virológica definitiva.

Pregunta 67.-R: 3 Pregunta 70.-R: 2


En general la hepatitis por virus B tiene una tasa de cronificación En general un título de anti S menor de 10 es muy bajo y, aunque
baja que oscila alrededor del 5% en adultos. La vía de adquisición no excluye que el paciente tenga una respuesta inmune porque no
no influye en la cronicidad, salvo en los niños que lo suelen adquirir es un inmunodeprimido, lo adecuado es plantearlo como un fallo
en el canal del parto y tienen un riesgo mucho más elevado de parcial en la respuesta y administrar inmunoglobulina específica y
padecer una forma crónica, llegar a cirrosis o quedar como portado- una dosis de recuerdo de la vacuna, siendo innecesaria la vacuna-
res. Por eso la respuesta correcta es la 3. ción completa. Es más seguro esta pauta que administrar únicamente
una dosis de recuerdo.
Pregunta 68.-R: 3
Los pacientes con VHB y hepatitis crónica o portadores crónicos, Pregunta 71.-R: 4
cuando sufren un episodio de hepatitis aguda debe pensarse en las El colapso multilobular es propio de las situaciones de fracaso
siguientes posibilidades: hepático fulminante. La presencia de un infiltrado mononuclear sólo
1. Que está sucediendo el proceso de seroconversión: la pérdida de nos permitirá establecer el diagnóstico de hepatitis persistente y no el
la capacidad replicativa del virus. Esta es la causa más frecuente de crónica activa. Es muy importante, porque en las situaciones de
de elevación de las transaminasas; en general es poco intensa y hepatitis persistente no está indicado el tratamiento antiviral, pues se
no suele cursar con complicaciones, excepto ligera ictericia. Se da un margen a la evolución espontánea a la curación. Para estable-
diagnostica porque el DNA viral se negativiza. cer el diagnóstico de hepatitis crónica activa se necesita, como míni-
2. Sobreinfección por un virus de la hepatitis A. Esta situación es de mo, demostrar cierto grado de necrosis (por eso es la respuesta co-
alto riesgo para desarrollar un fracaso hepático fulminante. Se rrecta), que puede llegar a establecer puentes de tejido colágeno en
recomienda que estos enfermos estén vacunados para el virus A, si las formas más avanzadas. La presencia de fibrosis masiva y nódulos
no han padecido previamente la infección. Esta situación se diag- de regeneración superan el diagnóstico de hepatitis crónica: ya se
nostica porque la IgM anti-VHA es positiva. trata de una cirrosis hepática.
3. Sobreinfección por el virus Delta. Esta situación es hoy infrecuente,
sólo se observa en hemofílicos y ADVP. Conlleva un riesgo elevado Pregunta 72.-R: 3
de hepatitis fulminante y, también, favorece que el virus Delta que- Se trata de una infección por el VHB, que indudablemente está
de en situación de cronicidad. Se diagnostica porque se observan, en fase replicativa, ya que el DNA, marcador específico de esta situa-
como en el presente caso, marcadores de infección aguda por virus ción, es positivo, por lo que la respuesta 1 es falsa. No hay una
D. El más importante y precoz de ellos es el antígeno Delta, que es situación de hepatitis aguda y el anti-Core es de tipo IgG únicamente.
positivo. En ningún caso puede ser una coinfección, ya nos lo La respuesta correcta es la número 3: se trata de una infección por la
aclara el enunciado, en el que se describe a un paciente diagnosti- mutante precore. Es fácil detectarla en el enunciado de la pregunta,
cado de infección crónica por el VHB y la coinfección es la situa- porque es un paciente con replicación, DNA positivo, pero con
ción de dos infecciones agudas simultáneas B y D (ver figuras). antígeno e negativo y anti-e positivo. Esto es lo contrario de lo que
sucede habitualmente con la cepa salvaje, que cuando replica es:
DNA virus B positivo, pero con antígeno e positivo y anti-e negativo.

Pregunta 73.-R: 2
Lamivudina es un antiviral muy eficaz para el tratamiento del
VHB. Sus indicaciones son:
1. En la cepa mutante precore (por eso la respuesta correcta es la 2).
Comentarios TEST

2. En VIH positivos.
3. En intolerancia o contraindicación al Interferón.

Tiene como ventajas que la administración es oral y tiene escasos


efectos adversos frente a Interferón; sin embargo se debe prolongar
su uso alrededor de un año, a diferencia del Interferón, que oscila
alrededor de los cuatro meses.
Lamivudina es una alternativa a Interferón, pero no está indicado
su uso combinado.

Pregunta 74.-R: 2
El objetivo primordial es lograr que deje de replicar el virus B, es
decir que se haga DNA negativo. Este proceso también se conoce
como seroconversión anti-e, ya que los pacientes pierden el antíge-
no e y desarrollan anticuerpos anti-e. El fundamento de este objetivo
es que, una vez pasado a no replicante, la progresión histológica de
la hepatitis queda detenida y, por tanto, se reduce casi por completo
el riesgo de desarrollar cirrosis hepática. Este objetivo se consigue
entre el 50-60% de los casos, pero sólo en un 15% se consigue
negativizar el antígeno de superficie y conseguir la inmunidad, sien-
do anti-s positivo. La mayoría de los pacientes no consiguen eliminar
por completo el virus y, aunque no sean replicantes, quedan como
portadores.

Pregunta 75.-R: 5
La enferma tiene, sin duda, una infección por virus C, puesto que
es anti-VHC positivo y el RNA es también positivo. Pero con normali-
Pregunta 68. Coinfección y sobreinfección por hepatitis D. dad de las transaminasas no se indica la biopsia hepática, porque a

9
DIGESTIVO
CTO Medicina 1ª Vuelta
pesar de que esté documentada la infección, no se va a prescribir El tratamiento es con ácido ursodesoxicólico, que mejora el pro-
tratamiento. Sólo se realiza biopsia a los pacientes con hipertransa- nóstico de la enfermedad y apenas tiene efectos adversos por lo que
minasemia prolongada. Sólo reciben tratamiento los pacientes en los es muy bien tolerado. Los esteroides no tienen utilidad y, en las fases
que la biopsia demuestre hepatitis crónica activa, no se trata de en- avanzadas de la enfermedad, cuando ya se ha desarrollado hiper-
trada a los que tienen hepatitis persistente en la histología. Por eso tensión portal está indicado el trasplante hepático.
este paciente precisa únicamente seguimiento evolutivo, que es la
respuesta correcta. Pregunta 81.-R: 5
El tratamiento, en caso de necesitarse, es actualmente la combi- En la valoración pronóstica de la cirrosis biliar primaria es fun-
nación de Interferón y Ribavirina, que consigue eficacia en práctica- damental conocer el nivel de bilirrubina, porque cuanto mayor
mente el 60% de los sujetos. Es un factor predictivo de eficacia tera- sea ésta, es decir más colestasis, se presume mayor territorio biliar
péutica comprobar que a las 12 semanas de tratamiento el RNA viral afecto. Los anticuerpos antimitocondriales son útiles, sobre todo,
se ha negativizado. Se considera éxito terapéutico cuando se cumple porque sugieren el diagnóstico. Igual sucede con el aumento poli-
el concepto de respuesta terapéutica sostenida: negativización per- clonal de IgM.
sistente del RNA seis meses después de concluir la terapia. Las hormonas tiroideas se solicitan porque es muy frecuente la
asociación a tiroiditis y, sobre todo, el desarrollo de hipotiroidismo
Pregunta 76.-R: 1 en estos enfermos. Los niveles de colesterol pueden ser útiles, puesto
En el tratamiento de la hepatitis crónica por VHB se utiliza Interfe- que están elevados en los enfermos con colestasis, no así los niveles
rón alfa, no gamma (respuesta correcta la 1). Este último se usa cuan- de triglicéridos, por eso es la respuesta que hay que señalar.
do hay enfermedades con deficiencia del sistema inmunológico,
como la enfermedad granulomatosa crónica. En relación con el tra- Pregunta 82.-R: 3
tamiento con Interferón en la infección crónica por el VHB, se han Esta enfermedad es infrecuente. Casi siempre suele ser ANCA pa-
objetivado factores de buen pronóstico: trón perinuclear positivo. Con mucha frecuencia existe el antecedente
1. El de mayor impacto predictivo es el de niveles bajos de DNA. previo de colitis ulcerosa o se asocia a ella a lo largo de su evolución.
2. Niveles entre 100 y 300 de transaminasas predicen una mejor Esta asociación es independiente y por lo tanto el desarrollo de la
respuesta que cuando oscilan entre 50 y 100. colitis ulcerosa no influye en el desarrollo y complicaciones de la
3. La respuesta es superior en mujeres e inferior en VIH positivos. colangitis esclerosante. Es más frecuente en varones, se asocia al HLA-
B8 y su diagnóstico se establece mostrando en la colangiografía retró-
Pregunta 77.-R: 1 grada endoscópica un patrón de estenosis irregulares, con dilatacio-
Actualmente las indicaciones de trasplante hepático han aumen- nes postestenóticas por todo el árbol biliar (ver tabla en página siguien-
tado y la edad sólo es un límite y no absoluto por encima de los 70 te). Además la biopsia hepática, aunque no ofrece el diagnóstico es de
años. Son factores de mal pronóstico para la recidiva: el genotipo interés, pues permite conocer si se ha establecido ya una cirrosis biliar
viral cuando es I, sobre todo si es b. Es muy probable que el hígado se secundaria, con lo que tiene de valor pronóstico este dato.
reinfecte por el VHC, pero la historia natural de esta infección, si no
existe otro factor asociado como el alcohol, suele ser la de una evo- Pregunta 83.-R: 2
lución lenta a la cirrosis en un período de tiempo de alrededor de 20 En el screening de los familiares de un paciente con hemocroma-
años y, además, si fuese necesario, se administra tratamiento antiviral. tosis se valora el índice de saturación de transferrina, que se conside-
Por eso la respuesta correcta es la número 1. ra patológico cuando es superior al 45% y la ferritina. El primero de
La presencia de crioglobulinas en sangre no establece ni excluye ellos es el más precoz en alterarse. Si estos marcadores bioquímicos
indicaciones de trasplante. Únicamente si se asocia a clínica de crio- están alterados, el siguiente paso es evaluar las mutaciones del gen
globulinemia está indicado el tratamiento secuencial con corticoi- HFE y, si se encuentra homozigoto o heterozigoto se establece el
des y, posteriormente, Interferón y Ribavirina. diagnóstico de hemocromatosis sin necesidad de una biopsia hepá-
tica y se indican a continuación las flebotomías periódicas.
Pregunta 78.-R: 3 La biopsia hepática es útil para el diagnóstico de aquellos casos
La presencia de ANA y ANCA en un enfermo con hipertransami- con sospecha clínica o bioquímica y estudio genético negativo. Ade-
nasemia persistente es muy sugestivo de hepatitis autoinmune. Esta más también se realiza en los casos con elevación persistente de las
situación obliga a excluir una infección viral y a practicar una biop- transaminasas para evaluar si hay fibrosis o cirrosis hepática.
sia hepática que demuestre una hepatitis, aunque no hay criterios
histológicos patognomónicos, por eso la respuesta 3 es la verdadera. Pregunta 84.-R: 1
Se trata de una hepatitis crónica autoinmnune tipo 1, ya que esta El gen anómalo está en el cromosoma 6. La herencia es autosómi-
presenta habitualmente ANA y/o antimúsculo liso positivos. La tipo 2 ca recesiva.

Comentarios TEST
se caracteriza por presentar estos anticuerpos negativos y ser anti- Entre el cuadro clínico destacan: cirrosis y sus complicaciones;
LKM positivo. Los ANCA, variedad perinuclear, pueden ser positivos miocardiopatía; pigmentación bronceada de la piel; diabetes melli-
en ambos tipos de hepatitis autoinmune. El tratamiento es con corti- tus que una vez establecida no mejora con las flebotomías; insufi-
coides y casi todos requieren un largo mantenimiento con azatiopri- ciencia exocrina del páncreas; artropatía generalmente con condro-
na una vez conseguida la remisión. El tacrolimus empleado en el calcinosis que no mejora con el tratamiento e hipogonadismo, que
trasplante hepático se está utilizando en algunos casos de hepatitis puede ser el cuadro de presentación en jóvenes por eso es la res-
autoinmune a nivel experimental. puesta correcta y que se debe no a una lesión genital sino hipofisaria.

Pregunta 79.-R: 1 Pregunta 85.-R: 2


El tratamiento de elección de un hepatocarcinoma es la extirpación La macroesteatosis hepática es una situación en la que predomi-
quirúrgica siempre que se pueda. Peores resultados ofrece el trasplante na el acúmulo de triglicéridos en el interior del hepatocito, con lo
hepático, pero pasa a ser de elección como en este caso si el paciente que éstos adquieren un aspecto hinchado y el hígado ecográfica-
tiene un grado funcional hepático avanzado, como es el grado C de mente presenta una ecogenicidad “brillante”. Suele cursar con un
Child, ya que en esta situación no toleraría la cirugía, pues le dejaría en ligero aumento de GOT o sin ninguna alteración. Suele ser reversi-
una situación de insuficiencia hepática grave. En las situaciones de le- ble, aunque recientemente se están describiendo algunos casos de
siones grandes y múltiples no se puede realizar cirugía y se intenta como cirrosis idiopática atribuibles a la evolución de esta entidad.
tratamiento paliativo para mejorar la supervivencia la alcoholización per- Las causas más frecuentes son la obesidad, diabetes mellitus tipo
cutánea. Cuando hay metástasis o trombosis de la porta extrahepática 2, el alcoholismo y el uso de corticoides. No debemos confundir esta
únicamente se utiliza el tratamiento sintomático. entidad con la esteatohepatitis microvesicular, que también se puede
producir en el alcohol y que se observa en el síndrome de Reye o en
Pregunta 80.-R: 5 el consumo de ácido valproico. Este cuadro es más grave y cursa
Presenta elevación predominante de la fosfatasa alcalina, supo- como una hepatitis aguda severa.
nemos que de origen hepático, por lo que presenta una colestasis.
Como la ecografía abdominal fue normal, no demostrándose por Pregunta 86.-R: 4
tanto dilatación de la vía biliar, es que la colestasis es intrahepática. Los cálculos de bilirrubina se producen porque pasa a la vía biliar
En este contexto los anticuerpos antimitocondriales positivos sugie- una cantidad de bilirrubina no conjugada superior a la tolerable.
ren fuertemente el diagnóstico de cirrosis biliar primaria. A continua- Esto sucede cuando se saturan sus mecanismos por:
ción estaría indicado una biopsia hepática para confirmarlo.

10
DIGESTIVO
1ª Vuelta CTO Medicina

Pregunta 82. Diferencias entre CBP y CEP.

CIRROSIS BIL IAR PRIMARIA COL ANGITIS ESCL EROSANTE PRIMARIA


Epidemiología M ujer. Va rón.
- Alt. inm unida d hum ora l y celula r. - Desconocida .
Patogenia
- Ac. a ntim itocondria les M 2. - Asocia ción HLA B8.
- I Cola ngitis destructiva .
Anatomía - II Gra nulom a s e infla m a ción periporta l.
- Destrucción y fibrosis de los conductos bilia res intra y extra hepá ticos.
Patológica - III Fibrosis septa l sin gra nulom a s.
- IV Cirrosis.
- Lo m á s frecuente ASTENIA Y PRURITO.
- Asintom á ticos dura nte m uchos a ños.
Clínica - Hepa toesplenom ega lia .
- Luego prurito, a stenia , ictericia , pérdida de peso.
- Lesiones por ra sca do.
- Lo m á s frecuente Sd. SECO.
Enfermedades - EII sobre todo COLITIS ULCEROSA.
- Escleroderm ia , hipotiroidism o.
asociadas - Fibrosis retroperitonea l o m edia stínica .
- ATR.
- Aum ento de FOSFATASA ALCALINA y resto de - Colesta sis crónica .
L aboratorio  enzim a s de colesta sis con tra nsa m ina sa s norm a les. - 1/3 hiperga m m a globulinem ia .
- Aum enta el colesterol e IgM . - 1/2 a um enta IgM , 65 % p-ANCA+.
- Sospecha clínica y la bora torio. - CPRE: DE ELECCIÓN.
Diagnóstico
- Confirm a ción: BIOPSIA. - Estenosis y dila ta ción de los conductos intra y extra hepá ticos.
Sintom á tico: Sintom á tico: el m ism o.
- Colestira m ina .
- Vita m ina s liposolubles.
Tratamiento Específico: Específico:
- Ac. ursodesoxicólico. - M etotrexa te.
- Colchicina , m etotrexa te. - Tra spla nte
- Tra spla nte.
- M ejor correla ción con el nivel de - 10 a ños de supervivencia tra s dia gnóstico.
Pronóstico
BILIRRUBINA. - Aum enta la incidencia de COLANGIOCARCINOM A.

1) Hiperaporte, como ocurre en los estados hemolíticos crónicos, res- geográficas. En tercer lugar con menor frecuencia está involucrada la
puesta correcta, o en el síndrome de Gilbert. Los cálculos son ne- microlitiasis y por último las causas idiopáticas u otras inhabituales en-
gros. tre las que se incluyen: el esfínter de Oddi hipertensivo, la fibrosis
2) Lesiones de la vía biliar, como en las colangitis crónicas o en las quística y fármacos como pentamidina, DDI o azatioprina.
cirrosis, siendo en esta ocasión cálculos de bilirrubina marrones.
Pregunta 90.-R: 4
Los cálculos de colesterol se producen porque se reduce la ab- El diagnóstico de pancreatitis crónica se establece, si el paciente
sorción de sales biliares, como sucede en la enfermedad de Crohn, o presenta síntomas, solicitando en primer lugar una Rx simple de ab-
aumenta la excreción hepática de colesterol, o hay una situación de domen para evaluar la presencia de calcificaciones. Si éstas se de-
hipomotilidad biliar como sucede en la obesidad y en la gestación. muestran el diagnóstico queda establecido. Si no se observan, enton-
ces se solicita una TAC, que es la prueba más sensible para observar
Pregunta 87.-R: 3 las calcificaciones pancreáticas. Si el TAC no es diagnóstico, enton-
Comentarios TEST

La bilis se vuelve más litogénica después de resecar el íleon o por ces se recurre a la CPRE, que es la prueba más sensible y específica,
una enfermedad ileal extensa porque no se reabsorben las sales bi- la que detecta los casos más precoces, pero que no se utiliza de
liares y como consecuencia aumenta la proporción en la vía biliar entrada porque es cruenta y tiene efectos secundarios. El test de la
del componente de colesterol sobre el de ácidos biliares. Esto provo- secretina diagnostica insuficiencia pancreática exocrina, pero es de
ca que el colesterol cristalice. En una bilis con cristales de colesterol, escasa utilidad para el diagnóstico de pancreatitis crónica.
ante cualquier situación de estasis en la vesícula, se formarán cálcu-
los por aposición de calcio sobre los cristales de colesterol. Por eso Pregunta 91.-R: 5
en la enfermedad de Crohn los cálculos son de colesterol. Entre los indicadores de gravedad el más precoz es la elevación de
los niveles séricos de la elastasa de los neutrófilos. Menos interés tiene
Pregunta 88.-R: 2 determinar la PCR (proteína C reactiva) o el péptido activador del
El diagnóstico de pancreatitis aguda se establece en un enfermo tripsinógeno urinario. Entre los indicadores radiológicos de gravedad
con dolor abdominal agudo por la elevación de la amilasa o la lipasa destacan los que se obtienen con un TAC con contraste, ya que más
por encima de tres veces el límite superior de la normalidad. En de un 50% de tejido necrótico confiere mal pronóstico.
ocasiones se sospecha el cuadro, pero las enzimas no son diagnósti- Clásicamente se han utilizado los criterios de Ransom (ver esquema
cas y, entonces, se requiere la realización de un TAC, que observan- en página siguiente), entre los que no está la determinación de PCR .
do las alteraciones radiológicas inflamatorias del páncreas, permite
establecer el diagnóstico de pancreatitis aguda. Pregunta 92.-R: 1
El interés de la ecografía en el manejo de la pancreatitis aguda se Una pancreatitis aguda leve o moderada requiere como trata-
dirige más a evaluar su etiología. Si se demuestra litiasis o microlitiasis miento:
biliar se establece en la ecografía que el diagnóstico etiológico de la 1. Analgesia, excepto mórficos porque contraen el esfínter de Oddi.
pancreatitis aguda es biliar. Si fuese necesario evaluar las complica- 2. Fluidoterapia i.v.
ciones de una pancreatitis o su pronóstico, el TAC ofrece más infor- 3. Dieta absoluta a sólidos y líquidos, al menos 3 días. Es muy impor-
mación que la ecografía. tante para evitar complicaciones diferir el inicio de la alimenta-
ción oral al menos hasta que el paciente esté asintomático, por
Pregunta 89.-R: 2 eso la respuesta correcta es la número 1. Sólo está indicado colo-
La litiasis biliar es la causa más frecuente de pancreatitis aguda, car una sonda nasogástrica de aspiración si hay vómitos secunda-
representando alrededor del 60%. El alcoholismo es la segunda cau- rios a un íleo paralítico asociado.
sa oscilando entre el 30 y 40% de los casos dependiendo de las áreas No está indicado administrar antibióticos.

11
DIGESTIVO
CTO Medicina 1ª Vuelta

Comentarios TEST
Pregunta 91. Actitud diagnóstica y terapéutica en la pancreatitis aguda.

Pregunta 93.-R: 1 la enfermedad, por eso es la respuesta correcta. En este caso la cole-
En el manejo de una pancreatitis aguda grave es muy importante cistectomía estaría indicada, pero en un segundo paso cuando se
evaluar mediante una punción si presenta infección del tejido necró- resuelva la pancreatitis aguda.
tico, porque en ese caso está indicada la necrosectomía quirúrgica. Si
no se demuestra infección, únicamente se toman medidas de soporte, Pregunta 95.-R: 1
antibióticos i.v. (preferentemente que perfundan bien en tejido pan- La causa más frecuente de pancreatitis crónica en nuestro medio
creático, como imipenem o las quinolonas). La somatostatina no ha es el alcoholismo crónico, oscilando alrededor del 70% de los casos.
demostrado su utilidad en el manejo de estos enfermos, que, además, Generalmente se inicia manifestándose como episodios de pancrea-
requieren nutrición parenteral o, si fuera posible, nutrición enteral titis aguda recurrente y, finalmente, como dolor crónico. Otras cau-
yeyunal, porque esta última parece reducir las tasas de infección de la sas como hipertrigliceridemia, hipercalcemia, fibrosis quística o se-
necrosis pancreática. cundaria a obstrucción de los conductos pancreáticos son menos
frecuentes.
Pregunta 94-R: 1
Se trata de una pancreatitis aguda de origen biliar que cumple Pregunta 96.-R: 2
criterios de gravedad. En este caso, además del manejo general de Además del dolor puede aparecer diabetes mellitus en los estadíos
una pancreatitis aguda grave, se ha comprobado que la papilotomía finales de la enfermedad. Esta diabetes tiene parecidas complicacio-
endoscópica realizada en las primeras 72 h. mejora el pronóstico de nes a las de la idiopática, excepto:

12
DIGESTIVO
1ª Vuelta CTO Medicina
1. Menor tendencia a la cetoacidosis
2. Como tiene poco glucagón, más hipoglucemias.

Además se desarrolla maldigestión, con esteatorrea, pérdida de


peso y malabsorción de las vitaminas liposolubles, pero también
déficit de B12, ya que la lipasa es necesaria para la absorción de esta
vitamina.
En ocasiones cursa con masas pancreáticas que al obstruir la vía
biliar causan ictericia.
Una complicación infrecuente es la fuga de líquido de un pseu-
doquiste o del propio Wirsung al interior del peritoneo, provocando
el cuadro de ascitis pancreática. Se diagnostica por una amilasa muy
elevada en líquido ascítico.

Pregunta 97.-R: 3
En una CPRE la presencia de una estenosis única en un enfermo
con dolor sugiere mucho más un carcinoma de páncreas que pan-
creatitis crónica. Las pancreatitis crónicas de cualquier tipo suelen
tener irregularidades múltiples, con estenosis y áreas de dilatación
postestenóticas.

Pregunta 98.-R: 4
Una vez conocido que un paciente tiene insuficiencia pancreá-
tica exocrina, bien mediante el test más fiable, como el de la
secretina, o con los de menor rentabilidad como el de pancreolauryl
o elastasa fecal, queda por determinar si hay que administrar enzi-
mas pancreáticas y, para ello, es suficiente con conocer el valor de
las grasas fecales (respuesta correcta), porque este es el marcador
que establece la indicación del tratamiento sustitutivo. De igual
forma la valoración del beneficio terapéutico se establece: con los
cambios en la clínica y la cuantificación de las grasas fecales que
demuestre desaparición o reducción de la esteatorrea.

Pregunta 99.-R: 1
Para el manejo de la ascitis pancreática es fundamental detectar el
punto de fuga del jugo pancreático entre los conductos pancreáticos
y el peritoneo. Para ello, es de elección la CPRE. Aunque la TAC
pueda mostrar un pseudoquiste es más difícil que detecte el punto
de fuga, algo crucial para establecer las indicaciones quirúrgicas.

Pregunta 100.-R: 4
El manejo del dolor en la pancreatitis crónica se realiza de forma
progresiva:
1. Analgesia convencional: paracetamol, metamizol, AINES.
2. Suplementos con enzimas pancreáticos en ocasiones alivian el
dolor.
3. Analgésicos de tercer nivel, excepto agonistas mórficos puros.
4. Dilataciones o colocación de prótesis por CPRE si hay obstruc-
ciones del Wirsung responsables del dolor.
5. Cirugía derivativa o incluso pancreatectomías parciales.
Comentarios TEST

Los antagonistas del TNF no tiene utilidad en la pancreatitis cróni-


ca, recordemos que para el examen MIR pueden estar indicados en
las fístulas refractarias de la enfermedad de Crohn y en casos de
artritis reumatoide refractarios al tratamiento convencional.

13

Potrebbero piacerti anche